7 - Hematology and Oncology

Editors: Schrier, Robert W.

Title: Internal Medicine Casebook, The: Real Patients, Real Answers, 3rd Edition

Copyright 2007 Lippincott Williams & Wilkins

> Table of Contents > Chapter 7 - Hematology and Oncology

function show_scrollbar() {}

Chapter 7

Hematology and Oncology

Paul A. Seligman

Acute Leukemia

  • What is the pathology of acute leukemia?

  • What are the primary classifications of acute leukemia, and why is this differentiation important?

  • P.271


  • What is the French, American, and British (FAB) classification of acute leukemia?

  • Are there any predisposing factors associated with acute leukemia?

  • What workup and other preparations should be done before initiating antileukemic therapy?

  • What are induction, consolidation, maintenance chemotherapy, and meningeal prophylactic therapy, and how do they differ in the treatment of acute lymphocytic leukemia (ALL) and acute nonlymphocytic leukemia (ANLL)?

  • What are the risks associated with antileukemic therapy, and what results can be expected?

Discussion

  • What is the pathology of acute leukemia?

    Acute leukemia is the abnormal clonal expansion of blood cell precursors. The abnormality may occur at different stages of maturation of the cell, and this explains the different types of leukemia. Acute leukemia is usually a rapidly progressive disease, although there are occasional patients whose disease remains stable for weeks or even months. In general, however, it is not the leukemic cells per se that cause the morbidity and mortality in this disorder, but a lack of normal blood cells, resulting in anemia, thrombocytopenia, and leukopenia. This is brought about by the leukemic cells crowding out the normal cells in the bone marrow. Other data suggest that especially myeloid leukemia cells have an inhibitory effect on normal marrow cells. This lack of normal cells may therefore lead to life-threatening hemorrhage and infection.

  • What are the primary classifications of acute leukemia, and why is this differentiation important?

    The primary classifications of acute leukemia are ALL and acute nonlymphocytic leukemia (ANLL, myeloid leukemia). The distinction is important because the therapy differs for each type (see answer to question 6). The overall ratio of ALL to ANLL is 1 : 6. ALL occurs most commonly in children, whereas ANLL more commonly affects adults.

  • What is the FAB classification of acute leukemia?

    The FAB classification (Table 7-1) is based largely on the morphologic and histochemical characteristics displayed by the leukemic cells, as well as on the nature of the cell surface antigens and cytogenetic features. This information may lead to changes in patient management, either by directing the course of therapy or by defining the prognosis better. Table 7-1 also includes molecular changes that may affect therapy, but more often affect response to therapy.

  • Are there any predisposing factors associated with acute leukemia?

    Certain genetic and environmental factors may predispose a person to acute leukemia. Many chromosomal alterations exist in the setting of the leukemias. The incidence of leukemia is increased in patients with congenital disorders associated with aneuploidy, such as Down syndrome, congenital

    P.272


    P.273


    agranulocytosis, celiac disease, Fanconi syndrome, and von Recklinghausen's neurofibromatosis.

    Table 7-1 The French, American and British (FAB) Classification of Acute Leukemia

          Associated
    FAB     Chromosome
    classification Description Comment Abnormalities
    ALL  
    L1 Small blasts with little cytoplasm, little cell-to-cell variation Most common morphology in childhood ALL 12 : 21
    L2 Larger cells with greater amount of cytoplasm, greater cell-to-cell variation; irregular nuclei with multiple nucleoli Most common morphology in adult ALL
    L3 Large cells, strongly basophilic cytoplasm; often with vacuoles; nucleoli often multiple Common in leukemia associated with Burkitt's lymphoma 8 : 14 (i.e., Burkitt's lymphoma)
    ANLL
    M1 Acute myelocytic leukemia: cells very undifferentiated with only occasional granules
    M2b Acute myelocytic leukemia: cells more differentiated with granules, and often with Auer rods 8:21a
    M3 Acute promyelocytic leukemia: hypergranular promyelocytes Often associated with disseminated intravascular coagulation, responds to differentiation agents 15:17a
    M4b Acute myelomonocytic leukemia: both monocytes and myelocytes predominate Often occurs with extramedullary infiltration (gingival hypertrophy, leukemia cutis, and meningeal leukemia) Inversiona 16
    M5b Acute monocytic leukemia: monoblasts with relatively agranular cytoplasm Usually affects children or young adults
    M6 Erythroleukemia: red blood cell precursors predominate, but myeloid blasts may also be seen Also called Di Guglielmo's syndrome
    M7 Megakaryocytic leukemia: extremely variable morphology; may be diagnosed with monoclonal antibodies to platelets Rare form of leukemia; very poor prognosis
    aThese karyotypes are generally considered to be more likely to respond to chemotherapy.
    bWhen M2, M4, and M5 leukemia occur after long-term myelodysplasia 11q 2; 3, monosomy 7 and other abnormal
    karyotypes suggest decreased response to chemotherapy.
    ALL, acute lymphocytic leukemia; ANLL, acute nonlymphocytic leukemia.

    Environmental factors implicated in the development of acute leukemia, particularly ANLL, include exposure to ionizing radiation and chemicals. Occupations and therapy that involve radiation exposure are known to increase the risk for acquiring acute leukemia. Chemicals, particularly the industrial use of benzene, and several therapeutic drugs (chloramphenicol, phenylbutazone, melphalan, chlorambucil, and others) are causal factors in acute leukemia. The findings from animal studies link certain viruses with acute leukemia; however, it is uncertain which viruses are actually an etiologic factor in human forms of leukemia, except for lymphomas caused by viruses that develop into a form of ALL.

  • What workup and other preparations should be done before initiating antileukemic therapy?

    The pretreatment evaluation should include the patient's medical and work history, especially the nature of any radiation or chemical exposure. A physical examination should include the patient's temperature, plus examination of the optic fundi, lymph node areas, oropharynx and gingivae, perianal area, and cranial nerves. Laboratory studies should consist of a complete blood count with differential (the physician should examine the smear), as well as a blood chemistry profile that includes the measurements of uric acid and lactate dehydrogenase (LDH). Bone marrow aspirates and biopsy specimens should be obtained, and investigations should include cytogenetic studies. A transfusion workup should include human lymphocyte antigen (HLA) typing. Lumbar puncture should be performed in all patients suspected of having ALL or ANLL-M4, and the cerebrospinal fluid specimen should be subjected to the usual studies, plus cytologic analysis. A dental examination should be performed.

    In addition, the patient's condition should be stabilized before antileukemic therapy is initiated. Hemorrhage and infection should be brought under control. Greatly elevated myeloblast counts (e.g., >50,000/mm3) that occur in the setting of ANLL can lead to pulmonary complications as well as fatal intracerebral leukostasis and hemorrhage. Cranial irradiation, hydroxyurea, and leukapheresis have all been used to decrease the numbers of circulating leukemic cells rapidly, and hence reduce the risk of complications. (Because of the physical properties of the lymphocytic leukemic cell, this is rarely a problem in patients with ALL.)

    Renal damage stemming from urate nephropathy may exist at the time of presentation or may occur with therapy, therefore urine alkalinization may prevent the need for dialysis. Patients should receive allopurinol (300 to 600 mg) for at least 24 hours before therapy to reduce the uric acid load, and this treatment should be continued until leukopenia and bone marrow hypocellularity have been achieved.

  • P.274


  • What are induction, consolidation, maintenance chemotherapy, and meningeal prophylactic therapy, and how do they differ in the treatment of ALL and ANLL?

    These are the phases of therapy used for acute leukemia. Induction therapy is usually the initial therapy and is intended to accomplish complete remission (that is, no signs or symptoms of disease, normal blood counts, and no evidence of leukemia, i.e., <5% blasts in the bone marrow). This therapy is usually administered on an inpatient basis, and is very toxic. Consolidation therapy is given after complete remission is achieved. It is similarly toxic, and consists of either the same drugs as those used in induction therapy or different ones. Its object is to reduce the now clinically undetectable leukemic cell mass as much as possible. Maintenance therapy is usually given on an outpatient basis and is less toxic, although complications of therapy can and do arise. This phase usually lasts for 2 to 3 years. Meningeal prophylactic therapy is given by means of lumbar puncture or through a reservoir placed under the scalp that cannulates the third ventricle. Its goal is to reduce the recurrence rate of leukemia in the central nervous system (CNS), which is considered a sanctuary site.

    All four therapy phases are used in ALL. In the treatment of ANLL, there is controversy over the use of maintenance therapy, although a second consolidation phase may be used. Meningeal prophylaxis is not used in the treatment of adult ANLL. However, CNS leukemia is more common in childhood ANLL, and prophylaxis is sometimes used in this setting. In general, the response to treatment and the prognosis are better in patients with ALL than in those with ANLL.

  • What are the risks associated with antileukemic therapy, and what results can be expected?

    As already noted, acute leukemia is usually a rapidly progressive disease that is fatal without therapy. Because the therapy itself is toxic, the mortality rate during induction therapy for ANLL may reach as high as 20%. Some toxicities are specific to the drug used, and these are not discussed here. Nearly all therapies provoke nausea and vomiting, which can be controlled with medications. More significantly, antileukemic therapy is intended to deplete the bone marrow, with subsequent repopulation by normal cells. During this period of depletion, the patient becomes severely thrombocytopenic and must be supported by platelet transfusions (given prophylactically at various intervals to keep the platelet count above 10,000) and, usually, also by red blood cell transfusions.

    Patients also become severely leukopenic, and this makes them very susceptible to infection. The typical signs and symptoms of infection (pus and purulent sputum) are often due to the actions of granulocytes, so infection is often subtle. The oral mucosa and perirectal areas are commonly overlooked sites of infection. Fever in a neutropenic patient must be considered infectious by origin, until proved otherwise. When this happens, examination and cultures should be carried out and broad-spectrum antibiotic therapy started quickly. Antifungal agents are usually added if no improvement is seen after 4 to 7 days of fever. The patient must be monitored carefully and treated for herpes virus infection because disseminated infection can be rapidly fatal.

    P.275


    If the leukemic cell burden is great, antileukemic therapy may precipitate the tumor lysis syndrome, caused by the rapid release of cell degradation products. It is characterized by hyperuricemia (causing urate nephropathy), hyperkalemia, hyperphosphatemia, and hypocalcemia. Advance recognition of patients at risk and subsequent treatment with vigorous hydration, allopurinol, and urine alkalinization 24 to 48 hours before the start of chemotherapy can usually prevent the syndrome. These patients must have their electrolyte, uric acid, phosphorus, calcium, and creatinine status repeatedly checked. Any metabolic abnormalities should be corrected and, if necessary, renal dialysis instituted early. Once the leukemic cell burden is decreased and degradation products cleared, the syndrome resolves.

    Most children with ALL respond to therapy and achieve long-term survival. Although 90% of adults with ALL experience complete remission with initial therapy, the median remission duration ranges from 48 to 60 months, depending on the study. Median survival is 3 to 5 years. However, approximately one third of all patients achieve long-term disease-free survival. Late recurrences are rare.

    Patients with ANLL face a worse prognosis. Approximately 75% experience complete remission, but most cases recur within 36 months. Of those who achieve complete remission, 20% to 25% show long-term disease-free survival. Bone marrow or stem cell transplantation with high-dose chemotherapy is often used, but is still under investigation as a therapy after the initial chemotherapy in ALL. The timing of transplantation (first remission, first relapse, or second remission), especially in ALL, is controversial. In ANLL, bone marrow transplantation (bone marrow rescue) with high-dose chemotherapy after a first remission has been associated with higher long-term survival rates. Older age (>40 years), use of unrelated donors, and evidence for residual disease at the time of transplantation reduce the efficacy of this treatment approach.

Case

A 63-year-old white man is seen in the emergency room with complaints of fever, fatigue, and malaise. He reports having intermittent epistaxis during the last week, mouth sores for the last 3 days, and a nonpruritic rash over his lower extremities, which was noted 24 hours before. He has experienced midchest pain for the last day, only on swallowing. He denies chemical, drug, or radiation exposure.

Physical examination reveals a temperature of 38.6 C (101.48 F). He has mild tachycardia, at 108 beats per minute. Head, eyes, ears, nose, and throat findings consist of a few petechiae over the soft palate. Multiple white plaques are seen on the oral mucosa, and there is hypertrophy of the gingivae. During examination of the skin, petechiae are found over the distal lower extremities. Other examination findings are normal. Specifically, no lymphadenopathy or hepatosplenomegaly are found. Other sites of possible infection, including the chest and perirectal area, are clear. The chest radiographic study is likewise normal.

Laboratory findings are as follows: white blood cell count, 17,200/mm3 with 2% polymorphonuclear leukocytes, 1% band forms, 16% lymphocytes, 4% monocytes, 5%

P.276


metamyelocytes, 4% basophils, and 68% blastocytes; hemoglobin, 11.1 g/dL; hematocrit, 32.6%; and platelets, 14,000/mm3. His electrolyte, blood urea nitrogen (BUN), creatinine, and aminotransferase levels are normal. His uric acid level is mildly increased at 9.2 mg/dL (normal, 3.5 to 8.0 mg/dL), as are his LDH level at 373 IU/L (normal, 30 to 220 IU/L). Examination of a peripheral blood smear reveals occasional nucleated red blood cells, few platelets, and many large cells containing finely reticulated nuclei, several nucleoli, cytoplasmic granules, and occasional Auer rods. Large cells with folded nuclei and large, prominent nucleoli are also seen.

  • What is the most likely diagnosis in this patient?

  • How is the absolute neutrophil count (ANC) calculated, and what is it in this patient?

  • Of what importance is the ANC?

  • Do the evaluation findings point to any specific infections?

  • What would you expect this patient's bone marrow to show?

  • Should a lumbar puncture be performed in this patient?

Case Discussion

  • What is the most likely diagnosis in this patient?

    Considering the results of this patient's complete blood count and peripheral blood smear, he has ANLL. The granular myelocytes and monocytes in the smear and the clinical evidence of extramedullary leukemic infiltration (gingival hypertrophy) point to a diagnosis of M4, or acute myelomonocytic leukemia. Examination of bone marrow specimens using special stains and chromosomal analysis can help confirm this diagnosis.

  • How is the ANC calculated, and what is it in this patient?

    To calculate the ANC, multiply the total white blood cell count by the percentage of polymorphonuclear leukocytes plus the percentage of band forms. In this case, the patient has 17,200 white blood cells, with 2% polymorphonuclear leukocytes and 1% band forms, or: 17,200 (0.02 + 0.01) = 516 absolute neutrophils.

  • Of what importance is the ANC?

    The ANC furnishes a rough estimate of the patient's ability to fight infection. A patient with an ANC of less than 500 is considered neutropenic and very susceptible to overwhelming infection. This patient, with an ANC of approximately 500, fever, and a presumed diagnosis of acute leukemia, falls into this category. Careful examination, together with cultures of blood, sputum, oral lesions, and other possible sites of infection, should be done quickly, and the patient started on broad-spectrum antibiotics immediately. Any delay in the workup or institution of antibiotics may result in overwhelming and possibly fatal infection. Cultures are often negative in neutropenic patients, although clinically they appear to be septic and respond to antibiotics.

  • Do the evaluation findings point to any specific infections?

    This patient complains of midchest pain on swallowing and physical examination reveals white oral plaques. A presumptive diagnosis of Candida esophagitis can be

    P.277


    made on the basis of these findings, and the patient should be started on antifungal agents as well as broad-spectrum antibacterial antibiotics. Neutropenic patients are susceptible to opportunistic infections, and candidiasis is very common in them.

  • What would you expect this patient's bone marrow to show?

    The bone marrow in this patient with ANLL would likely exhibit hypercellularity, with cellular elements often constituting 90% or more of the marrow. The numbers of red blood cell precursors and megakaryocytes will be decreased. The morphology may be normal, or there may be dyserythropoiesis (asynchronous maturing of the nuclear and cytoplasmic elements). The marrow will primarily show a monotonous pattern of cells similar to those seen in the peripheral smear. Flow cytometry should show cell surface markers indicative of immature myeloid cells with monocytoid characteristics. The chromosome analysis may show an abnormality such as monosomy 7 (especially if the patient had myelodysplasia), but will not show the abnormalities associated with, for example, M3 leukemia (Table 7-1). Recent studies suggest complex karyotypes in patients older than 60 years, that is, three or more aberrations have decreased response to therapy and based on comorbid factors these patients should be considered for investigational therapy or supportive care.

  • Should a lumbar puncture be performed in this patient?

    This patient has a presumptive diagnosis of acute myelomonocytic leukemia. Lumbar punctures are routinely done in cases of ALL and ANLL-M4 because these leukemias are associated with meningitis. Nevertheless, any patient with acute leukemia and symptoms of meningitis or cranial nerve palsies should undergo a diagnostic lumbar puncture, regardless of the leukemic type.

    However, the platelet count in this patient is only 14,000/mm3, and lumbar punctures should not be performed when the platelet count is less than 50,000/mm3 because of the risk of hemorrhage. Therefore, platelet transfusions must be given before attempting lumbar puncture to bring the count to 50,000/mm3 or more.

Suggested Readings

Baccarani M, Carbelli G, Amadori S, et al. Adolescent and adult acute lymphoblastic leukemia: prognostic features and outcome of therapy a study of 293 patients. Blood 1982;60:677.

Bennett JM, Young ML, Anderson JW, et al. Long-term survival in acute myeloid leukemia. Cancer 1997;8:2205.

Burnett A, Goldstone AH, Stevens RMF, et al. Randomized comparison of addition of autologous bone-marrow transplantation to intensive remission: results of MRC AML 10 trial. Lancet 1998;351:700.

Farag SS, Archer KJ, Mrozek K, et al. Pretreatment cytogenetics add to other prognostic factors predicting complete remission and long-term outcome in patients 60 years of age or older with acute myeloid leukemia: results from Cancer and Leukemia Group B 8461. Blood 2006;108:63.

Gale RP, Hoelzer D. Acute lymphoblastic leukemia. New York: Wiley-Liss, 1990.

Koeffler HP. Syndromes of acute nonlymphocytic leukemia. Ann Intern Med 1987;107:748.

P.278


Anemia

  • What is the definition of anemia, and what is the differential diagnosis based on the mean corpuscular volume (MCV)?

  • Why is it important to examine the peripheral blood smear, and what are the many diagnostic erythrocyte abnormalities and corresponding clinical conditions?

  • What is a reticulocyte, and how is the reticulocyte count used to characterize an anemia? What is the reticulocyte index, how is it calculated, and how is it used in the differential diagnosis of anemia?

  • What is the difference between - and -thalassemia, how are they distinguished clinically, and how is electrophoresis useful?

  • What is sickle cell anemia, and how is it manifested clinically? What is the sickle cell trait, and how is it manifested clinically?

Discussion

  • What is the definition of anemia, and what is the differential diagnosis based on the MCV?

    Anemia is usually defined as an abnormally low hematocrit or hemoglobin concentration, and occurs when the rate of erythrocyte loss exceeds the rate of erythrocyte production. The differential diagnosis of anemia depends on whether the MCV is low, high, or normal. Table 7-2 lists the various possible diagnoses for each of these categories. Sometimes with mild anemia, a diagnosis may be entertained if the MCV is in the high or low range of normal.

  • Why is it important to examine the peripheral blood smear, and what are the many diagnostic erythrocyte abnormalities and corresponding clinical conditions?

    Peripheral blood smear examination can reveal erythrocyte abnormalities that point to the correct diagnosis of the anemia. Echinocytes, or burr cells, for example are seen in uremia and pyruvate kinase deficiency. Elliptocytes are the abnormal erythrocytes seen in patients with hereditary elliptocytosis.

    P.279


    Nucleated red cells are found in the setting of stress or hematologic disease with bone marrow involvement. Schistocytes or fragments occur in patients with microangiopathic hemolytic anemia. Sickle cells are found in the setting of sickle cell anemia. Spherocytes occur in immune-mediated hemolytic anemia and hereditary spherocytosis. Target cells form in the presence of liver disease and iron deficiency; they also occur after splenectomy.

    Table 7-2 Differential Diagnosis of Anemia Based on Mean Corpuscular Volume (MCV)

    Low MCV Normal MCV High MCV
    -Thalassemia Acute blood loss Alcohol abuse
    -Thalassemia Aplastic anemia Aplastic anemia
    Iron deficiency Chronic disease Cobalamin deficiency
    Lead poisoning Combination of macrocytic and Folate deficiency
    Sideroblastic anemia microcytic causes Hemolysis
      Hemoglobinopathy Hypothyroidism
      Hemolysis Liver disease
      Iron deficiency Myelodysplastic syndromes

  • What is a reticulocyte, and how is the reticulocyte count used to characterize an anemia? What is the reticulocyte index, how is it calculated, and how is it used in the differential diagnosis of anemia?

    A reticulocyte is a young circulating red blood cell that exhibits basophilia under vital staining. The reticulocyte count is used to characterize the bone marrow's attempt to compensate, if at all, for the anemia present. The reticulocyte index (Table 7-3) is a more useful means of characterizing anemia because it is determined by correcting the reticulocyte count for the hematocrit, assuming a normal hematocrit is 45%. This correction is necessary because reticulocytes are counted per 1,000 red blood cells.

    An index of less than 2 is found in the setting of the hypoproliferative anemias. These consist of disorders of heme or globin synthesis, such as iron deficiency, anemia stemming from chronic disease, lead poisoning, sideroblastic anemias, and , , and other thalassemias; megaloblastic anemias resulting from cobalamin or folate deficiency; myelodysplastic syndromes; aplastic anemias; and other metabolic causes, such as renal insufficiency and hypothyroidism.

    Hyperproliferative anemias are associated with a reticulocyte index greater than 2. These anemias arise as the result of acute blood loss; nutrient replacement, such as cobalamin, folate, or iron replacement, but before the resolution of anemia; both hereditary and acquired hemolysis; and primary or secondary polycythemia.

    Automated reticulocyte counts introduced for general clinical practice are more accurate than hand counts and automatically calculate the reticulocyte index. These automated values also include the total number of reticulocytes, a measurement that might be helpful when obtaining serial values.

    Table 7-3 The Reticulocyte Index

    Hematocrit (%) Correction Factor
    45 1.0
    35 1.5
    25 2.0
    image
    Correction of the reticulocyte index for shift cells: shift cells-newly released erythrocytes.
    Newer automated systems that will become available will give corrected reticulocyte count and a reticulocyte maturation index to account for shift cells.

  • P.280


  • What is the difference between - and -thalassemia, how are they distinguished clinically, and how is electrophoresis useful?

    The -thalassemias constitute abnormalities of the gene, or genes, responsible for the synthesis of the chain of hemoglobin. Humans contain four genes for this purpose and each is responsible for approximately a fourth of the chains synthesized. Any combination of from one to four of these genes may be missing. Thalassemia is unapparent clinically when only one gene is missing, and this is called 1-thalassemia. This defect exists in up to 30% of the American black population. If two of the genes are missing, the entity is referred to as 2-thalassemia. These patients are usually asymptomatic, although their hematocrit and MCV may be slightly low. This defect affects approximately 2% of African Americans. When three of the genes are lacking, the patient exhibits the phenotype of -thalassemia (Hemoglobin H disease) with a low hematocrit and MCV, and -chain tetramers or hemoglobin H is found in the red blood cells. When all four genes are missing, the result is usually a stillborn infant with hydrops fetalis.

    -Thalassemia is the most common form of thalassemia in the Southeast Asian population.

    The -thalassemias consist of abnormalities of the gene, or genes, responsible for the chain of hemoglobin, and they cause insufficient -chain synthesis. This leads to the formation of -chain tetramers and inclusions of this hemoglobin attached to the plasma membranes of erythrocytes, resulting in hemolysis. Patients with heterozygous -thalassemia exhibit a modest decrease in their hematocrit values and a marked decrease in their MCVs. Patients with homozygous -thalassemia have severe anemia and low MCVs. They require transfusion, and complications may arise stemming from the excess accumulation of iron.

    Electrophoresis may be used to suggest the diagnosis of -thalassemia in patients missing three genes, and thereby having sufficient fast-migrating hemoglobin H ( l- and 2-thalassemia traits may not be detected). The precise number of missing genes can be determined in hybridization studies through the use of a complementary DNA probe.

    The findings yielded by hemoglobin electrophoresis are usually diagnostic in the setting of -thalassemia. Because -chain synthesis is normal in these patients, the other hemoglobins seen in adults, including hemoglobin A2 and F, are increased in a compensatory manner. Therefore, patients with heterozygous -thalassemia would have elevated hemoglobin A2 and F levels with hemoglobin A present. Patients with homozygous -thalassemia would have no hemoglobin A and markedly elevated hemoglobin F and A2 levels.

  • What is sickle cell anemia, and how is it manifested clinically? What is the sickle cell trait, and how is it manifested clinically?

    Sickle cell disease is the most commonly recognized clinically significant hemoglobinopathy. It stems from a substitution of valine for glutamic acid in the chain of hemoglobin and can be diagnosed by electrophoresis. Sickle cell anemia results when both chains are abnormal. Sickled cells should be evident on

    P.281


    a peripheral blood smear. Hemoglobin S is less soluble than normal hemoglobin at a low oxygen tension, causing the hemoglobin molecules to crystallize, which deforms the red blood cells. These misshapen cells greatly increase the blood viscosity, which leads to small-vessel occlusion and hence pain and organ infarctions, specifically stroke as well as pulmonary, renal, and bone infarction.

    Sickle cell disease may be manifested by a variety of crises: Pain is the most common symptom, and is thought to be secondary to red blood cell sludging and infarction. Splenic sequestration and dactylitis are common in children, but rare in adults. Aplastic anemia is uncommon, but is typically associated with infections, and is anticipated when reticulocyte counts decrease in the face of worsening of anemia. Megaloblastic anemia is usually secondary to folate deficiency, and arises because abnormal cells have a shortened life span. This increases the turnover of red blood cells and places an increased demand on folate stores. Sickle cell patients who enter with pain crisis or chest syndrome are at risk for multiorgan failure. When it becomes apparent that liver, kidney, and/or pulmonary function are declining, these patients should be considered for exchange transfusion.

    The sickle cell trait is almost always asymptomatic because only one of the two chains is abnormal. It can also be diagnosed by electrophoresis, and this is most important for the purposes of genetic counseling.

Case 1

A 42-year-old man is seen by his primary care physician because of a rectal urgency. On sigmoidoscopy, a mass is located at 8 cm. He undergoes resection to remove the mass and after surgery he receives adjuvant chemotherapy and undergoes pelvic radiation therapy. After he completes therapy, he returns to his primary care physician 6 months later with complaints of fatigue and dyspnea on exertion. As part of the evaluation, a complete blood count is obtained and reveals the following findings: white blood cell count, 3.9 109/L; hemoglobin, 8.2 g/dL; hematocrit, 24.4%; MCV, 86 fL; reticulocytes, 1%; and platelets, 450,000/mm3. The patient has a serum iron content of 23 g/dL, a total iron-binding capacity of 256 g/dL, and a ferritin level of 10 ng/mL.

  • What is the likely cause of this patient's anemia, and how would you evaluate him further?

  • If the patient is iron deficient, why is his MCV 86 fL?

  • On the basis of the patient's iron status, what treatment should be prescribed, and how should therapy be monitored?

Case Discussion

  • What is the likely cause of this patient's anemia, and how would you evaluate him further?

    The cause of this patient's anemia is likely multifactorial. However, the ferritin level below 12 ng/mL and the percentage transferrin saturation (total iron-binding Fe/capacity) below 10% are both diagnostic for iron deficiency. He should receive oral iron supplementation, but he should be evaluated for a gastrointestinal source of

    P.282


    blood loss (e.g., recurrent tumor, second primary cancer, or some other nonmalignant source).

    The patient may also be anemic and leukopenic due to the extensive exposure of the bone marrow to radiation during pelvic radiation therapy. This bone marrow damage may be compounded by the concomitant chemotherapy treatment.

    Finally, the possibility of other contributory factors, such as folate and cobalamin deficiency, should also be investigated.

  • If the patient is iron deficient, why is his MCV 86 fL?

    The MCV may be normal in the settings of early iron deficiency, although the red blood cell distribution width is high under these circumstances. The MCV may also be normal in iron-deficiency anemia complicated by another nutritional deficiency, such as folate or cobalamin deficiency. In this patient, the MCV is likely higher than expected as a result of his recent chemotherapy treatment that is associated with inhibition of DNA synthesis.

  • On the basis of the patient's iron status, what treatment should be prescribed, and how should therapy be monitored?

    The patient has iron deficiency. Ferrous sulfate (300 mg three times a day) provides 180 mg of elemental iron per day, which should normalize the hematocrit over the course of several months. The hematocrit should increase by 1% to 3% each week and his reticulocyte count should also increase significantly with this treatment.

    The status of the absorption of oral iron can be easily demonstrated by determining the fasting serum iron level before and 3 to 4 hours after the ingestion of a single 300-mg tablet of ferrous sulfate. If normal, the level should rise by a minimum of two times the baseline (fasting) value. If the patient has decreased iron absorption, that is, due to inflammation block that decreases absorption, he should be treated with intravenous iron.

Case 2

A 67-year-old woman is seen for complaints of mild memory loss and fatigue. On evaluation, she is found to have an anemia, which is characterized by the following laboratory values: white blood cell count, 5,200/mm3; hemoglobin, 9.1 g/dL; hematocrit, 26.9%; MCV, 101 fL; reticulocytes, less than 1%; and platelets, 154/mm3. Her serum cobalamin level is 260 pg/mL and her folate, thyroid-stimulating hormone, and liver function tests are normal. The patient does not abuse alcohol, and her peripheral blood smear is unrevealing.

  • How would you further evaluate this patient's anemia?

  • On the basis of the laboratory results so far, what test, or tests, might be helpful in diagnosing the cause of this patient's anemia?

  • Why might such a patient be deficient in cobalamin?

Case Discussion

  • How would you further evaluate this patient's anemia?

    Serum cobalamin and folate levels should be determined. In addition, a search for both ethanol abuse and liver disease should be undertaken and hypothyroidism

    P.283


    ruled out. If none of these is found to be a likely cause, other reasons for the anemia (refractory or aplastic anemia) should be explored. A peripheral blood smear should be examined for possible clues such as hypersegmented polymorphonuclear leukocytes (seen in cobalamin deficiency) or target cells (seen in liver disease).

  • On the basis of the laboratory results so far, what test, or tests, might be helpful in diagnosing the cause of this patient's anemia?

    This patient likely has cobalamin deficiency, although her serum cobalamin level of 260 pg/mL is within the normal range. Because studies have shown that such deficiency results in methylmalonic aciduria and homocystinemia, these metabolic substrates should be measured in this patient. Other testing that might be considered includes a Schilling test or measurement of anti intrinsic factor-blocking antibodies.

  • Why might such a patient be deficient in cobalamin?

    There are various causes of cobalamin deficiency. It can stem from the ingestion of insufficient animal protein, as seen in true vegetarians. Failure to release cobalamin from food binders or failure to secrete intrinsic factor results in pernicious anemia. Failure to absorb the intrinsic factor cobalamin complex in the distal ileum, as occurs in patients who have undergone an ileal resection or who have regional enteritis, can also lead to cobalamin deficiency. Rare causes are abnormal or absent enzymes or transport proteins, and nitrous oxide abuse.

Suggested Readings

Akarsu S, Taskin E, Yilmaz E, et al. Treatment of iron deficiency anemia with intravenous iron preparations. Acta Haematol 2006;116:51.

Beutler E, Lichtman MA, Colter BS, et al., eds. Hematology, 6th ed. New York: McGraw-Hill, 2000.

Wintrobe MM, ed. Clinical hematology, 10th ed. Philadelphia: Lea & Febiger, 1998.

Bleeding Disorders

  • What are the major divisions of the coagulation system?

  • What are the general screening tests for evaluating each of the major divisions of the coagulation system?

  • What common disorders are associated with each of the major divisions of the coagulation system?

  • What are the clinical manifestations of various bleeding disorders?

  • What workup is indicated for a bleeding patient?

  • What therapies are available for the management of bleeding disorders?

Discussion

  • What are the major divisions of the coagulation system?

    The coagulation system is quite complex, but can be viewed as consisting of at least three major components: the vascular endothelium, the blood

    P.284


    coagulation proteins (both those that promote clotting and those that lyse clots by means of the fibrinolytic system), and the platelets. The coagulation cascade represents a series of proteins that, when initiated, forms a fibrin clot. A simple outline of the cascade is shown in Table 7-4. Complex issues such as the exact mechanisms by which anticoagulants, such as protein C and protein S, function and how factor VII may activate factor IX are not completely understood.

    Table 7-4 The Intrinsic and Extrinsic Pathways of the Coagulation System

  • What are the general screening tests for evaluating each of the major divisions of the coagulation system?

    Vascular endothelial integrity can be assessed using the bleeding time. In this test, a nick is made in the skin under standardized conditions, and the time to cessation of bleeding is measured.

    The blood coagulation proteins are usually evaluated by in vitro studies using the patient's citrate-anticoagulated plasma. This is done by adding back various components of the coagulation cascade to the patient's plasma to induce clot, and the procedure is standardized against plasma from an individual with normal plasma coagulation components. The two most common tests for doing

    P.285


    this are the prothrombin time (PT) and the partial tissue thromboplastin time (PTT). The PT measures the extrinsic pathway of the coagulation cascade, and this is done by adding tissue thromboplastin to the patient's plasma. If there is a deficit in any of the common pathway components or factor VII, the clotting time is prolonged abnormally. The PTT measures the intrinsic and common pathways; a deficit in the common or intrinsic pathway proteins results in a prolonged PTT. A third, less commonly used, screening test is the thrombin time, which measures only the last step in the cascade the conversion of fibrinogen to fibrin and is done by adding thrombin to the patient's plasma. Therefore, if the patient has too little fibrinogen or a dysfunctional fibrinogen protein, the time is prolonged. Finally, each of the components of the cascade, including factors I to XIII, can be assayed directly to evaluate for deficits.

    Platelets can be evaluated both quantitatively (by the platelet count) and functionally. Platelet function can be assessed by the bleeding time; qualitatively defective platelets do not form an adequate platelet plug and the bleeding time is prolonged. In addition, platelets can be analyzed in vitro for their aggregability using platelet stimulants (e.g., ristocetin).

  • What common disorders are associated with each of the major divisions of the coagulation system?

    The vascular endothelium may be fragile in the setting of several acquired conditions, including vasculitis and long-term steroid use. This is important to realize because it may cause the bleeding time to be prolonged despite normal platelet number and function.

    Deficits in the blood coagulation proteins may be congenital or acquired. The most common congenital disorders consist of deficiencies in factor VIII (hemophilia A) or factor IX (hemophilia B, or Christmas disease), which are inherited in an X-linked manner. Another common congenital disorder is von Willebrand's disease, in which there is a deficit in von Willebrand's factor. This factor is bound to factor VIII and is necessary for both platelet function and for clotting to take place by the intrinsic pathway.

    Deficiencies in various factors can be acquired when their production is antagonized, as occurs with sodium warfarin (Coumadin; DuPont Pharma, Wilmington, DE) therapy, a substance that inhibits the production of activated vitamin K dependent factors (factors II, VII, IX, X, and protein C and S). Another common situation that causes deficiencies in various factors is liver disease; because the liver is the site for the synthesis of nearly all the coagulation factors, severe liver disease results in deficient production of factors. Malnutrition, malabsorption, and liver disease can all lead to a deficit in vitamin K, with a subsequent deficit in the vitamin K dependent factors. Finally, the overwhelming consumption of all factors can result in a coagulopathy, as occurs in disseminated intravascular coagulation (DIC).

    The platelet population can be depressed because of either underproduction or excessive destruction. Underproduction occurs as a consequence of bone marrow suppression (brought about by chemotherapy, infections, drugs, or infiltration with other cells, such as occurs in the setting of leukemia or

    P.286


    cancer). Excessive destruction can occur in the setting of an enlarged spleen (sequestration), bleeding (consumption) or consumptive disorders (DIC or thrombotic thrombocytopenic purpura/hemolytic uremic syndrome), and on an autoimmune basis [idiopathic thrombocytopenic purpura (ITP)].

    Qualitative defects can be congenital, but are more often acquired and due to drug exposure (aspirin, nonsteroidal antiinflammatory drugs, and some antibiotics) or uremia.

  • What are the clinical manifestations of various bleeding disorders?

    Although any of the bleeding disorders may result in excessive hemorrhage associated with such events as surgical procedures, trauma, or gastrointestinal bleeding, each displays some characteristic features. Vascular fragility is typically associated with subcutaneous ecchymoses. Plasma coagulation protein deficiencies in patients with hemophilia are associated with spontaneous soft tissue and joint bleeds. Other plasma factor deficiencies, as well as platelet deficits, are associated with diffuse ecchymoses (cutaneous and soft tissue). Platelet deficits are also manifested by petechiae (small capillary hemorrhages in mucosal surfaces and areas of increased hydrostatic pressure, such as the ankles and feet) and purpura (larger areas of hemorrhage). Von Willebrand's disease is unique in that it may present with both soft tissue bleeding (factor VII deficiency) and mucosal bleeding (platelet dysfunction).

  • What workup is indicated for a bleeding patient?

    Evaluation of the bleeding patient begins with a good history taking. It needs to be determined if the condition is of long standing or is new. Questions about previous bleeding episodes (nosebleeds, bruising, menstrual flow, bleeding with trauma, surgery, and delivery) as well as family history are vital for determining the nature of the disorder. A careful drug history, including over-the-counter drug use, must be taken. The patient's medical history and a review of symptoms may reveal evidence of autoimmune disorders or intercurrent illness.

    Physical examination is important in evaluating the sites of bleeding (cutaneous, mucosal, soft tissue, or joint bleeding sites, as well as petechiae). An enlarged spleen and evidence of liver disease (e.g., spiders or hemangiomata) or malnutrition should be sought, and the patient's overall medical condition should be assessed.

    A screening for bleeding disorders should include a platelet count, PT, and PTT; if any of these results are abnormal or if there is evidence of mucosal bleeding, determination of a bleeding time may also be indicated.

    If the PT or PTT is prolonged, the next step in the evaluation should be a 1:1 mix in which the patient's plasma is mixed with normal plasma and the PT and PTT are determined again. If the patient is deficient in some factor, the normal plasma partially corrects this deficiency and the PT or PTT are corrected to a normal value. If an inhibitor to a particular factor is present, this inhibitor also blocks the action of the normal plasma, and the PT or PTT are not corrected. The most common inhibitor is the lupus anticoagulant, which is seen in the presence and absence of autoimmune disease; it is usually

    P.287


    associated with an elevated PTT that is not corrected with a 1:1 mix. It is associated with an increased risk of clotting, not bleeding.

    If the platelet count is very low (20,000/mm3) and the PT and PTT are normal, a bone marrow biopsy may be indicated to determine whether there are adequate platelet precursors in the bone marrow. If platelet precursors are absent, an underproduction state exists; if precursors are present, this implies that the low platelet count stems from peripheral destruction. Using the detection of antiplatelet antibodies as evidence for the autoimmune destruction of platelets is not reliable because some normal people have antiplatelet antibodies without peripheral destruction, whereas the titers in people with ITP may be low.

  • What therapies are available for the management of bleeding disorders?

    Blood components can be used to correct deficiencies in the divisions of the coagulation system. Fresh frozen plasma contains various percentages of each of the coagulation proteins and can be used when more than one factor is deficient (e.g., vitamin K dependent factors). Cryoprecipitate contains von Willebrand's factor, fibrinogen, and factor VIII, but is most commonly used in people with an acquired fibrinogen deficiency (e.g., DIC and liver disease). Because of the risk of viral infection (it is pooled from multiple donors), cryoprecipitate is no longer used as frequently for patients with mild hemophilia and von Willebrand's disease. Instead, desmopressin (DDAVP) is now used in the treatment of these diseases, as well as in the platelet dysfunction associated with uremia and other qualitative defects. This drug works by stimulating the release of von Willebrand's factor (factor VIII) from the endothelium. There are also specific heat-treated factor concentrates for factors VIII and IX, which can be used in the management of hemophilia.

    Quantitative platelet problems caused by underproduction, as well as some consumptive states such as uncontrolled bleeding, can be treated with platelet transfusions. This is often futile in the setting of autoimmune destruction until the autoimmune process is arrested; in fact, platelet transfusion may accelerate destruction by stimulating the immune system. The usual initial treatment for ITP is with high-dose prednisone, followed by splenectomy if the prednisone fails to block the immune destruction. Transfusing platelets into a patient who has uremia or who is taking a drug that renders his or her own platelets dysfunctional is also futile because the transfused platelets quickly become affected as well.

Case 1

A 47-year-old white man comes to the emergency room complaining of hematemesis and a 4-day history of abdominal pain and passing black, tarry stools. He gives a history of peptic ulcer disease that is linked to heavy alcohol use, and this was associated with one previous episode of bleeding. He denies the use of any medications, including over-the-counter medicines, and denies a family history of bleeding. On review of the systems, he describes some increased bruising during the last 2 to 3 months. On physical examination he is found to be jaundiced and in moderate distress; alcohol is smelled on his breath. His skin is remarkable for scattered ecchymoses and spider angiomas. His liver

P.288


span is 15 cm and there is some tenderness plus a palpable spleen tip. The patient is continuing to pass melena and vomit bright red blood.

The following initial laboratory values are found: white blood cell count, 4,500/mm3 with a normal differential; hemoglobin, 6.0 g/dL; hematocrit, 18%; platelets, 87,000/mm3; aspartate aminotransferase (AST), 95 mU/mL (normal, 0 to 35 mU/mL); alanine aminotransferase (ALT), 40 mU/mL (normal, 0 to 38 mU/mL); total bilirubin, 3.5 mg/dL (normal, <1.0 mg/dL); and alkaline phosphatase, 450 mU/mL (normal, 0 to 125 mU/mL).

  • How would you proceed with the evaluation of this patient's bleeding problem?

  • What blood products, if any, would you give this patient?

  • What other medicines, if any, would you give this patient to manage his bleeding?

  • What factors may be contributing to this patient's low platelet count?

Case Discussion

  • How would you proceed with the evaluation of this patient's bleeding problem?

    While emergency medical management of his bleeding is being provided through the placement of a nasogastric tube, together with the intravenous administration of fluids for blood pressure support as needed and typing and crossmatching in preparation for the administration of packed red blood cells, this patient with apparent chronic liver disease needs to have his coagulation status evaluated. Both the PT and PTT should be determined promptly and measurement of the fibrinogen level should be considered because it can be decreased in the setting of chronic liver failure. In this case, if the PT and PTT prove to be elevated, as expected, there is probably little reason for a 1:1 mix in this acutely ill patient because a deficiency state is very likely.

  • What blood products would you give this patient, if any?

    If his PT or PTT proves to be elevated, the best blood product for replacing the deficient factors is fresh frozen plasma. In addition, if his fibrinogen level is measured and found to be less than 100 mg/dL, cryoprecipitate may also be indicated. Finally, it may become necessary to administer platelets if his count falls below 20,000/mm3 in the face of active bleeding.

  • What other medicines, if any, would you give this patient to manage his bleeding?

    If history and physical examination findings are consistent with alcoholism and liver disease, vitamin K should also be given.

  • What factors may be contributing to this patient's low platelet count?

    His low platelet count may stem from multiple causes. First, the platelet count can fall in the face of massive bleeding (consumption). Second, he may be chronically underproducing platelets owing to either chronic alcohol suppression of the bone marrow or folic acid deficiency. Finally, he has an enlarged spleen, which may be sequestering his platelets.

Case 2

A 35-year-old Hispanic woman presents to the emergency room complaining of a nosebleed that has persisted for several hours. She denies a history of previous bleeding,

P.289


although she has noticed some increased bruising during the last week and the appearance of a small, purplish rash on her feet and ankles. She denies any excessive bleeding with the delivery of her three children and has not undergone any surgical procedures. She denies taking aspirin, although she has taken acetaminophen for relief of a mild backache, and is on no other medications. On review of her symptoms, she denies arthralgias, arthritis, fevers, cold symptoms, or other infectious symptoms; she has been in good health until now. On examination, she is found to be well developed and in no distress. There is some fresh as well as dried blood obscuring the nasal mucosa; she has no conjunctival hemorrhages but does have palatal petechiae. Her spleen is not palpable but there is a petechial rash around both ankles. Her nosebleed requires nasal packing for control.

The following initial laboratory values are found: white blood cell count, 6,700/mm3 with a normal differential; hemoglobin, 14.2 g/dL; hematocrit, 42.2%; MCV, 85 3; platelets, 5,000/mm3; PT, 11.5 seconds (control, 12 seconds); and PTT, 28 seconds (control, 28.5 seconds).

  • What would you do next to evaluate this patient's bleeding?

  • What results would you expect from the further evaluation of this patient's bleeding?

  • What therapy would you institute in this patient?

Case Discussion

  • What would you do next to evaluate this patient's bleeding?

    With the normal coagulation findings and complete blood count, except for the platelet count, and the absence of other physical findings such as an enlarged spleen, a bone marrow biopsy is not essential to evaluate for megakaryocytes. Some clinicians may choose to treat for presumptive ITP and evaluate the patient in 24 hours.

  • What results would you expect from the further evaluation of this patient's bleeding?

    Her clinical picture is consistent with that of ITP, and, in this setting, an adequate bone marrow specimen would show an increased or normal number of megakaryocytes. If the physician chooses to treat the patient empirically for ITP (see the following text), the patient should have significant improvement (i.e., platelet count 20,000 with less incidence of bleeding) in 24 hours.

  • What therapy would you institute in this patient?

    Platelet transfusions would not be helpful in this patient and might even accelerate the destructive process. Prednisone treatment (60 to 100 mg per day) should be initiated once bone marrow findings confirm the diagnosis or if the patient is treated empirically.

Case 3

You are asked to consult on the case of a 65-year-old white man with a history of severe rheumatoid arthritis, who has cervical spine instability that now requires orthopaedic stabilization. The preoperative laboratory results are as follows: white blood cell count, 10,000/mm3 with a normal differential; hemoglobin, 12 g/dL; hematocrit, 36%; MCV,

P.290


86 fL; platelets, 190,000/mm3; PT, 12 seconds (control, 11.5 seconds); PTT, 52.2 seconds (control, 32.5 seconds); and bleeding time, 10.5 minutes (normal, 0 to 9.5 minutes).

The patient denies any bleeding history, and had undergone a right knee replacement in the past without difficulty. He has taken large doses of aspirin in the past, but is currently on a nonsteroidal agent and takes no other medicines. There is no family history of bleeding disorders. On examination, he exhibits the sequelae of severe chronic rheumatoid arthritis, with deformed joints of the hands. He has no significant skin lesions. His spleen is not palpable and his liver is not enlarged.

  • What further preoperative evaluation would you do to reassure the surgeon that intraoperative hemostasis is adequate?

  • What blood products, if any, would you use in this patient?

  • What changes, if any, would you make in this patient's medications?

Case Discussion

  • What further preoperative evaluation would you do to reassure the surgeon that intraoperative hemostasis is adequate?

    The patient's main coagulation abnormalities include a slightly prolonged bleeding time and an elevated PTT. His medications include a nonsteroidal antiinflammatory agent, which can reversibly affect platelet function; this is the most likely source of his mildly increased bleeding time. In the absence of a bleeding history and if no emergency circumstances prevail, a 1:1 mix of his elevated PTT is indicated. His history of a chronic inflammatory condition is a strong indicator to have his lupus anticoagulant level determined.

  • What blood products, if any, would you use in this patient?

    If a 1:1 mix does not correct in response to normal plasma, this indicates the presence of an inhibitor. His clinical picture is consistent with a lupus anticoagulant, which is actually associated with a risk of clotting, not bleeding, so no blood products are indicated. If his 1:1 mix does correct, implying a deficiency state, then specific assays of factor levels, including factors VIII and IX, may be necessary to identify the specific deficiency. This is highly unlikely in the absence of clinical bleeding.

    This patient's slightly prolonged bleeding time does not require any intervention.

  • What changes, if any, would you make in this patient's medications?

    His nonsteroidal medication should be stopped for at least 5 to 7 days before the spine stabilization procedure to allow normal platelet function to return. Immediately before surgery, his bleeding time should be checked again to confirm this return to normal.

Suggested Readings

Beutler E, Lichtman MA, Colter BS, et al., eds. Hematology, 5th ed. New York: McGraw-Hill, 1995.

Wintrobe MW. Clinical hematology, 9th ed. Philadelphia: Lea & Febiger, 1993.

P.291


Breast Cancer

  • What is the incidence of breast cancer?

  • What is the natural history of breast cancer?

  • What are the risk factors for breast cancer?

  • Of what does the screening for breast cancer consist?

  • What is the TNM classification, and what are the stages of breast cancer?

  • What are the prognostic indicators associated with breast cancer?

  • What is the difference between modified radical mastectomy and lumpectomy plus radiation therapy in the treatment of stage I and II breast cancer, and what are the indications for each?

  • What is the role for adjuvant chemotherapy in the treatment of breast cancer?

  • Of what does the treatment of node-negative breast cancer consist?

  • What is the purpose and the underlying principles of endocrine manipulation in the treatment of metastatic breast cancer?

  • What is the role of systemic chemotherapy in the treatment of metastatic breast cancer?

Discussion

  • What is the incidence of breast cancer?

    Breast cancer is the most common neoplasm in women, with an incidence that continues to rise and currently stands at 1 in 10 women. The incidence rises dramatically with age.

  • What is the natural history of breast cancer?

    Breast cancer is considered to be a systemic disease from the time of diagnosis, regardless of the stage. The average doubling time varies from 23 to 500 days. Therefore, a 1-cm tumor may have existed for 2 to 17 years before diagnosis.

    Despite local control, affected patients continue to die at a rate faster than that seen in age-matched control subjects for the first 30 years after treatment. In addition, patients dying from any cause are found to have evidence of tumor at autopsy. The most common sites of distant metastases are the bone, liver, and lung.

    Paraneoplastic conditions that may be associated with breast cancer include hypercalcemia, neuromuscular disorders, dermatomyositis, acanthosis nigricans, and hemostatic abnormalities.

    Common secondary malignancies in patients with breast cancer consist of cancer in the opposite breast, ovarian cancer, and colorectal carcinoma.

  • What are the risk factors for breast cancer?

    High-risk factors (threefold or greater increase) for the development of breast cancer are:

    • Age greater than 50 years

    • Previous cancer in one breast, especially that occurring premenopausally

    • P.292


    • Breast cancer in the family, although the risk varies depending on whether the disease was in a first-degree family member, was unilateral or bilateral, and occurred premenopausally or postmenopausally: bilateral and premenopausal disease carries an 8.8 times greater risk; bilateral and postmenopausal disease carries a 5.4 times greater risk; unilateral and premenopausal disease carries a 3 times greater risk; and unilateral and postmenopausal disease carries a 1.5 times greater risk

    • Parity. Women who are nulliparous or who were first pregnant after 31 years of age have a three to four times increased risk

    • Ductal carcinoma in situ carries a 30% risk of becoming invasive

    • Certain forms of benign breast disease are associated with an increased risk of cancer Gross cystic disease with lesions exceeding 3 mm, multiple intraductal papillomas, and atypical ductal hyperplasia are considered premalignant

    Intermediate-risk factors (1.2- to 1.5-fold increase) for the development of breast cancer consist of menstruation (either early menarche or late menopause); oral estrogen with progesterone therapy; alcohol consumption; diabetes mellitus; history of cancer of the uterus, ovary, or colon; and obesity.

  • Of what does the screening for breast cancer consist?

    Women older than 20 years should perform breast self-examination every month. Premenopausal women should examine their breasts 5 to 7 days after the end of their menstrual cycle, and postmenopausal women should do this on the same day of every month.

    Women should have their breasts examined by a physician every 2 to 3 years between the ages of 20 and 40 years and annually thereafter. The American Cancer Society and other agencies have recommended that a baseline mammogram should be obtained between the ages of 35 and 39 years, with mammograms obtained every 1 to 2 years in women aged 40 to 49 years and then yearly after the age of 50 years.

  • What is the TNM classification, and what are the stages of breast cancer?

    The TNM (primary tumor, regional nodes, metastases) classification and the various stages of breast cancer are outlined in Table 7-5.

  • What are the prognostic indicators associated with breast cancer?

    Table 7-6 summarizes the 5- and 10-year survival statistics associated with the various TNM stages. These statistics do not take into account results of adjuvant chemotherapy, but are useful in designing trials using adjuvant chemotherapy.

    The patient's hormonal status also has a bearing on her prognosis, in that estrogen- and progesterone receptor-positive tumors possess a 70% to 85% chance of responding to hormonal therapy; those women with only one-receptor positivity have exhibited a slightly lower response rate to hormone manipulation. Women with receptor-negative tumors do not respond to hormone manipulation.

    P.293


    Table 7-5 The TNM Classification of Breast Cancer

      Disease Extent  
    Stage Groupinga Primary Tumor (T) Lymph Nodes (N)b Distant Metastases (M) TNM Classification
    0 Noninvasive carcinoma in situ; Paget's disease of the nipple (Tis) Homolateral axillary nodes negative (N0) None Tis N0 M1
    I Greatest dimension 2 cm(T1)c Homolateral axillary nodes negative (N0) None T1 N0 M0
    II Greatest dimension >2 cmand 5 cm (T2)c Homolateral axillary nodes positive but not .xed (N1) T1 N1 M0
    T2 N0 or N1 M0
    IIIA Greatest dimension >5 cm(T3)c Homolateral axillary nodes positive and .xed to one another, skin, or chest wall (N2) None T1 N2 M0
    T2 N2 M0
    T3 N0 2 M0
    IIIB Any size with (T4)c satellite skin nodules, skin ulceration, fixation to skin or chest wall, or edema of breast, including peau d'oranged Supraclavicular or infraclavicular nodal involvement; edema of the arm with or without palpable axillary lymph nodes (N3) None T4 any N M0
    Any T N3 M0
    IV Any size Any status Present Any T any N M1
    aThe American Joint Committee recognizes two stage groupings: postoperative-pathologic (presented in this table) and clinical-diagnostic.
    bThe clinical-diagnostic stage grouping subdivides movable homolateral axillary lymph nodes into N1a nodes not considered to contain tumor (approximately 33% are histologically positive); and N1b nodes considered to contain tumor (approximately 25% are histologically negative).
    cT0 indicates no tumor demonstrable in breasts; T1, T2, and T3 include tumor fixation to underlying pectoral fascia or muscle, which does not change the classification of lesions. (In flammatory breast cancer is classified as a separate entity and is not included in T4.)
    dSkin dimpling and nipple retraction do not affect staging classification.
    Adapted from the American Joint Committee for Cancer Staging and End-Results Reporting, 1983

  • What is the difference between modified radical mastectomy and lumpectomy plus radiation therapy in the treatment of stage I and II breast cancer, and what are the indications for each?

    Although mastectomy controls local disease, it may have a devastating psychological impact on both the patients and their families; therefore, surgical techniques designed to preserve the breast are warranted. In 1976, the National

    P.294


    Surgical Adjuvant Breast Project (NSABP) began a randomized trial comparing total mastectomy, segmental mastectomy, and segmental mastectomy plus radiation. All 1,843 patients underwent axillary node dissection. The 16-year follow-up revealed that lumpectomy in patients with tumors less than 4 cm in diameter and with free surgical margins is an appropriate form of therapy in stage I and II breast cancer. In addition, irradiation plus lumpectomy markedly decreases the likelihood of local recurrence. Local recurrence, even 10 years postlumpectomy, does not affect overall survival.

    Table 7-6 Prognostic Indicators for Breast Cancer

    Prognostic Indicators 5 y (%) 10 y (%)
    Clinical stage    
      0   >90   90
        I   80   65
        II   60   45
        IIIA   50   40
        IIIB   35   20
        IV and inflammatory breast cancer   10   5
    Tumor size (cm)        
        <1      80
        3 4      55
        5 7.5      15
    Axillary nodes        
        None positive   80   65
        1 3 positive   65   40
        >3 positive   30   15

  • What is the role for adjuvant chemotherapy in the treatment of breast cancer?

    The lymph node status is the most important prognostic indicator in this disease. Patients with positive nodes are at a high risk for local recurrences as well as metastatic disease.

    A prospective, randomized trial showed that the addition of AC [doxorubicin (Adriamycin) and cyclophosphamide (Cytoxan)] to the treatment protocol improves the 10-year overall survival in both premenopausal and postmenopausal women. The addition of a taxane to this regimen provides a small but statistically significant improvement in disease-free survival. If the tumor is hormone receptor positive, adding antiestrogen treatment to the chemotherapeutic regimen has been shown in multiple trials to increase disease-free survival, and overall survival benefit has been shown.

  • Of what does the treatment of node-negative breast cancer consist?

    Some patients thought to be node negative in the past have been found to be node positive by careful analysis using techniques such as sentinel node biopsy. The treatment of node-negative breast cancer is still controversial. Originally,

    P.295


    women with negative nodes were thought to have a very good prognosis, but 30% were still found to be dying of the disease. Giving therapy to all such patients is not without hazard; therefore, it would be of great value to have indicators that could predict who would be good candidates for treatment. Tests can predict aggressive tumors such as increased cells in the S phase of the cell cycle and the presence of growth factor markers such as Her2 neu. Other factors, such as the size of the primary tumor, the histologic grade, and hormone receptor status, may also be influential. Most studies now suggest that patients with tumors larger than 1 cm should be offered adjuvant chemotherapy. Patients with a low-grade 1.3 cm tumor that is estrogen receptor (ER) and progesterone receptor (PR) positive and Her2 neu negative would show far less benefit from chemotherapy than the same size tumor that is high grade and both ER and PR negative. Her2 neu-positive tumors benefit most from chemotherapy and the addition of monoclonal antibody against the protein (trazatinmib), particularly if the tumors are larger than 1 cm or show evidence for aggressive disease. Tamoxifen in premenopausal patients and aromatase inhibitors in postmenopausal patients have been shown to protect patients from the development of breast cancer and are effective in preventing recurrence in patients with small tumors that are node negative but positive for estrogen and/or progesterone receptors.

  • What is the purpose and the underlying principles of endocrine manipulation in the treatment of metastatic breast cancer?

    It has been known for many years that there is an interrelationship between the ovaries and the breasts. Patients with locally recurrent breast cancer have exhibited a dramatic response to bilateral oophorectomy. More recently, with the ability to identify estrogen receptors in breast tissue, it was natural for antiestrogen therapy to be used for the treatment of breast cancer. The first trials of hormonal agents were conducted in patients with metastatic disease, and they proved that these agents were not only efficacious but also well tolerated, with weight gain being the only major side effect. Tamoxifen can increase risk of uterine cancer and blood clots; these side effects are not increased with aromatase inhibitors.

    Because of their success in the management of advanced disease, hormonal agents have been instituted as adjuvant therapy and chemoprevention agents. Although the finding of estrogen receptor positivity constitutes the greatest advantage, many women are negative for receptors, or their status is unknown.

  • What is the role of systemic chemotherapy in the treatment of metastatic breast cancer?

    Patients are candidates for chemotherapy if their disseminated disease is highly aggressive, they are hormone receptor negative, or they fail to respond to endocrine manipulation. There are several variations of combination chemotherapy regimens containing cyclophosphamide, methotrexate, 5-fluorouracil, doxorubicin, paclitaxel, carboplatin, and other agents. The use of single-agent chemotherapy with a new drug added after one fails is just as efficacious as the use of combination chemotherapy, unless the extent of disease requires a more rapid response for quality of life issues.

P.296


Case 1

A 35-year-old white woman with a family history of breast cancer discovers a lump in her right breast. The lump is confirmed on physical examination and a mammogram is then obtained. A 1.7-cm lesion is identified and sampled for biopsy. Pathologic analysis of the biopsy tissue reveals an infiltrating ductal carcinoma. The patient elects to undergo lumpectomy with sentinel lymph node sampling followed by axillary node dissection. Two of nine lymph nodes are positive, and estrogen and progesterone receptor studies are negative. The histologic grade of the tumor is 3/3, and the percentage S phase measured Ki-67 monoclonal antibody, is 18.5%.

The patient receives four cycles of AC, taxol, then local irradiation. She has no evidence of disease and is seen every 3 months for follow-up.

  • What is this patient's TNM classification and stage?

  • Was lumpectomy an appropriate treatment?

  • Does this patient have poor prognostic indicators?

Case Discussion

  • What is this patient's TNM classification and stage?

    This patient has a T1 lesion because her primary tumor was less than 2 cm. Her nodal status is N1 because two of the nodes showed tumor infiltration but were not palpable at presentation, and her metastasis status is graded as M0 because no metastases were found. Therefore, she has stage II disease.

  • Was lumpectomy an appropriate treatment?

    Evidence suggests that lumpectomy is an alternative to mastectomy in the management of stage II disease. Because of her two positive nodes, radiation therapy to the axilla is also recommended to lessen her increased potential for local recurrence. The finding of two positive nodes also makes her a candidate for more aggressive systemic chemotherapy to limit the chance for development of distant metastases.

  • Does this patient have poor prognostic indicators?

    This patient has several poor prognostic features: her age of 35 years; a high-grade tumor morphology together with a high-percentage S phase; negativity for both receptors; and the positive nodes. Her chance of surviving 10 years with no systematic treatment is approximately 35%, with systematic treatment she has a greater than 50% 10-year survival. New studies suggest that if she were Her2 neu positive her survival without systematic treatment is less than 30%, but the addition of trastumabib to her chemotherapy may actually improve survival compared with Her2 neu negative patients.

Case 2

A 62-year-old woman was first seen 12 years ago because of a 4-cm left breast mass. Biopsy results revealed adenocarcinoma, and the patient underwent a modified radical mastectomy and axillary node dissection. Two of 22 nodes were positive, and the tumor was positive for estrogen and progesterone receptors. The patient was placed on chemotherapy followed by tamoxifen therapy.

P.297


She did well until 6 years ago, when right hip pain developed. A bone scan revealed the presence of metastatic disease in her spine, ribs, and right hip. She was given anastrozole (Arimidex; Zeneca Pharmaceuticals, Wilmington, DE), an aromatase inhibitor used in postmenopausal patients as hormonal agent. Fourteen months later, pain occurred in her left shoulder and she became increasingly lethargic. A restaging evaluation showed progressive bone scan findings and hypercalcemia. She was started on chemotherapy and bisphosphonates were instituted to treat her hypercalcemia acutely. The patient received six cycles of chemotherapy, with subsequent stabilization of her disease; however, 22 months later, her disease progressed rapidly. She was treated with two other chemotherapeutic regimens, but after initial stabilization of her disease with each treatment, she died 36 months later.

  • What was this patient's original TNM classification and stage?

  • Is her clinical course typical of breast cancer?

  • Was a second hormonal agent warranted?

  • What was the cause of her hypercalcemia, and how should it be treated?

Case Discussion

  • What was this patient's original TNM classification and stage?

    The patient originally had a T2 lesion because her tumor was 4 cm, her nodal status was N1 because axillary dissection revealed seven positive nodes, and her metastasis status was M0 because no obvious metastatic lesions were discovered. Taken together, she originally had stage III disease.

  • Is her clinical course typical of breast cancer?

    Breast cancer is considered a chronic disease based on the hypothesis that micrometastases exist at the time of diagnosis. This theory is supported by the observation that women with early-stage breast cancer still exhibit an increased risk of dying of their disease despite curative intent, for 20 years.

  • Was a second hormonal agent warranted?

    The best predictor of hormonal response is a response to a previous hormonal agent. In this case, the patient theoretically responded to tamoxifen (based on delay in recurrence); therefore, another hormonal agent was appropriate, and this produced 14 months of further response.

  • What was the cause of her hypercalcemia, and how should it be treated?

    Two general mechanisms can bring about hypercalcemia in a patient with cancer: (a) tumor cells in direct contact with bone can induce an osteolytic mechanism; and (b) tumor cells can secrete humoral substances that activate osteoclasts.

    The first mechanism primarily operates in breast cancer. Acute intervention requires bisphosphonates with fluid diuresis. However, the patient must first be hydrated adequately before diuresis is started, because dehydration only worsens the hypercalcemia. Treatment of the underlying cause should then be instituted, as was done in this patient with chemotherapy. Bisphosphonates not only help treat hypercalcemia but, with continual use, also improve symptoms caused by breast metastases to bone and may improve survival.

P.298


Suggested Readings

Clark GM, Dressler LG, Owens MA, et al. Prediction of relapse or survival in patients with node-negative breast cancer by DNA flow cytometry. N Engl J Med 1989;320:627.

Early Breast Cancer Trialists' Collaborative Group. Polychemotherapy for early breast cancer: an overview of the randomized trials. Lancet 1998;352:930.

Fisher B, Costantino JP, Wickerham DL, et al. Tamoxifen for prevention of breast cancer: report of the National Surgical Adjuvant Breast and Bowel Project P{-}1 Study. J Natl Cancer Inst 1998;90:1371.

Fisher B, Redmond C, Poisson R, et al. Eight-year results of randomized clinical trial comparing total mastectomy and lumpectomy with or without irradiation in the treatment of breast cancer. N Engl J Med 1989;320:822.

McGuire WL. Adjuvant therapy for node-negative breast cancer. N Engl J Med 1989;320:525.

Olivotto IA, Bajdik CD, Ravdin PM, et al. Population-based validation of the prognostic model ADJUVANT! for early breast cancer. J Clin Oncol 2005;23:2716.

Chronic Myelogenous Leukemia

  • What is the definition of chronic myelogenous leukemia (CML)?

  • What is the etiology of CML?

  • What is the pathogenic mechanism responsible for CML?

  • What is the epidemiology of CML?

  • What are the clinical characteristics of CML?

  • What are the laboratory findings encountered in the setting of CML?

  • What are the cytogenetic and biochemical abnormalities found typically in patients with CML?

  • What is the treatment for CML?

  • What is the prognosis in patients with CML?

Discussion

  • What is the definition of CML?

    CML is a hematopoietic stem cell disease characterized by anemia, extreme blood granulocytosis, granulocytic immaturity, basophilia, often thrombocytosis, and splenomegaly.

  • What is the etiology of CML?

    The etiology of CML is unknown, but exposure to ionizing radiation has been found to increase the risk of CML above the expected frequency in certain populations. Some of these major populations are (a) the Japanese exposed to radiation from the Nagasaki and Hiroshima atomic bomb explosions; (b) the British with ankylosing spondylitis treated with spinal irradiation; and (c) women with uterine cervical carcinoma who require radiation therapy. The frequency of CML (as well as acute leukemia) in these populations is significantly greater than that expected for comparable unexposed groups. Chemical leukemogens have not been identified as causative agents of CML.

  • P.299


  • What is the pathogenic mechanism responsible for CML?

    CML results from the acquired (somatic mutation) malignant transformation of a single stem cell whose potency dominates hematopoiesis in the affected person, with the involvement of erythropoiesis, neutrophilopoiesis, eosinophilopoiesis, basophilopoiesis, monocytopoiesis, and thrombopoiesis. Several observations suggest that some lymphocytes may be derived from the primordial malignant cell as well, thereby placing the culprit lesion closer, if not in the pluripotential stem cell. The exact mechanism that causes the transformation to take place has not been fully elucidated, but the Ph1 (Philadelphia) chromosome has been implicated. The hematopoietic cells contain a reciprocal translocation between chromosomes 9 and 22 in more than 90% of patients. This leads to an overtly foreshortened long arm of one of the chromosome 22 pairs. Chromosome 9 contains the c-abl gene at band 34; chromosome 22 has the break point cluster region (bcr) and c-sis genes at band 11. The c-abl gene from chromosome 9 is transported to the chromosome 22 bcr, which is the Ph1 chromosome. As a consequence of these events, a new gene is formed, the bcr-abl gene, which codes for a new protein through the formation of a new messenger RNA. In some uses the chromosomal abnormality is not evident but the bcr-abl gene is identified by in situ by hybridization. This new protein is a phosphoprotein with a molecular weight of 210,000 (DaP210 bcr-abl) and possessing tyrosine kinase activity. Its abnormal activity presumably alters the response of the hematopoietic stem cell so that it continues to proliferate rather than being under the control of hematopoietic growth factors.

  • What is the epidemiology of CML?

    CML accounts for approximately 2% of all cases of leukemia and the associated mortality rate is approximately 1.5 per 100,000 population per year. The disease occurs slightly more often in men, but its manifestations and course are similar for both sexes. Approximately 10% of the cases occur in people between 5 and 20 years of age, and CML accounts for approximately 3% of all the childhood leukemias.

  • What are the clinical characteristics of CML?

    The disease is characterized by three phases: (a) a chronic phase, (b) an accelerated phase, and (c) a blast crisis.

    The most frequent complaints seen during the chronic phase include easy fatigability, loss of a sense of well being, decreased tolerance to exertion, anorexia, abdominal discomfort, early satiety, weight loss, and excessive sweating. The symptoms are vague, nonspecific, and gradual in onset. Physical examination may detect pallor and splenomegaly.

    Uncommon presenting signs and symptoms of CML include hypermetabolism that simulates thyrotoxicosis, acute gouty arthritis, priapism, tinnitus, stupor, left upper quadrant and left shoulder pain as a consequence of splenic infarction and perisplenitis, diabetes insipidus, and acute urticaria, which is associated with hyperhistaminemia.

    In some patients in this phase, the disease is discovered when blood cell counts are determined during a routine medical examination. The symptoms

    P.300


    and signs of the disease and the laboratory findings typically remain stable, and the duration of this phase is variable. Usually it lasts approximately 4 years, but it can last from weeks to many years before transforming to the accelerated phase.

    In most cases of CML, the patient's disease eventually changes to a more aggressive, symptomatic, and troublesome form (the accelerated phase) that responds poorly to therapy that formerly controlled the chronic phase. This metamorphosis is often gradual and manifested by refractory splenomegaly; extramedullary tumor masses; changes in the blood, bone marrow, and differential cell counts; and new cytogenetic abnormalities. The onset of fever without infection, weakness, night sweats, weight loss, arthralgias, and bone or left upper quadrant pain may occur before there is laboratory evidence of the accelerated phase. These laboratory abnormalities include a decrease in the hemoglobin content with increasing red blood cell abnormalities, an abrupt increase or fall in the white blood cell count without treatment, and an increase in the number of blast or immature cells. Thrombocytosis or thrombocytopenia and an increase in the number of basophils or eosinophils are also seen.

    The blastic phase can be manifested by an extramedullary blast infiltration or by a bone marrow blast crisis.

    An extramedullary blast crisis is the first manifestation of the accelerated phase in approximately 10% of patients, and this principally involves the lymph nodes, serosal surfaces, skin and soft tissue, breasts, and the CNS. Bone involvement may lead to severe pain, tenderness, and radiographic changes. The CNS involvement is usually meningeal and may be preceded by headache, vomiting, stupor, cranial nerve palsies, and papilledema; it is associated with an increase in the number of cells and the protein level, as well as the presence of blast cells in the spinal fluid.

    Acute leukemia, the blast phase, develops in most patients with CML, and this can take from days to years to occur after the diagnosis of CML depending on the effectiveness of initial treatment. The signs and symptoms are fever, hemorrhage, bone pain, and lymphadenopathy, as well as the other manifestations already cited. The blastic transformation is usually myeloblastic or myelomonocytic, but can be erythrocytic or lymphoid in nature. Special staining techniques, biochemical assays, or monoclonal antibody determinations are needed to identify the type of transformation once the patient is in the blastic phase. Patients usually die within weeks to months. The median survival in patients in the myeloid blast crisis is approximately 6 to 12 months, and that in patients in the lymphoid blast crisis is 12 months, with survival beyond 2 years unusual. Severe infections, hemorrhage, and organ dysfunction, especially of the liver and kidney, are among the leading causes of death.

  • What are the laboratory findings encountered in the setting of CML?

    The diagnosis of CML can be made on the basis of the hematologic findings, specifically those yielded by the blood count and the blood smear. Common findings are a decrease in the hematocrit; the presence of nucleated red blood cells in the circulation; a leukocyte count that is always elevated,

    P.301


    often exceeding 1,000 109/L; the presence of all stages of granulocyte development in the blood with a generally normal appearance; and a blast cell prevalence ranging from 0.5% to 5%. Myelocytes, metamyelocytes, and band forms account for approximately 40%. The number of basophils is increased, as is the total absolute lymphocyte count (mean, approximately 15 109/L). In addition, the platelet count is elevated in approximately 50% of patients at the time of diagnosis; platelet counts more than 1,000 109/L are not unusual; and neutrophil alkaline phosphatase activity is low or absent in more than 90% of patients. The defects in white cell adhesion, emigration, and phagocytosis are mild and compensated for by high neutrophil concentrations, and therefore do not predispose patients in the chronic phase to infections. Platelet dysfunction can occur but is not associated with spontaneous or exaggerated bleeding, as with other myeloproliferative disorders.

    In terms of the morphologic findings, the bone marrow is markedly hypercellular and hematopoietic tissue takes up 75% to 90% of the marrow volume. Granulopoiesis is dominant, with a granulocytic erythroid ratio of between 10 and 30:1 (normal, 2 to 4:1). Erythropoiesis is usually decreased, the megakaryocytes are normal or increased in number, and the population of eosinophils and basophils may be increased.

  • What are the cytogenetic and biochemical abnormalities typically found in patients with CML?

    The Ph1 chromosome, designated t(9;22)(q34;q11), is present in more than 90% of patients with CML. During the blast phase, most patients exhibit additional chromosome abnormalities, usually a +8, the gain of a second Ph1 chromosome, or rarely a chromosome loss (-7).

    Variant Ph1 chromosome translocations occur in approximately 5% of patients and usually consist of complex rearrangements. Every chromosome is involved except the Y chromosome. There is a small group of patients with CML who do not have the Ph1 chromosome, but virtually all patients have an abnormal chromosome 22 with bcr rearrangements. The characteristic biochemical abnormalities consist of an increase in the uric acid level, an increase in the serum level of cobalamin-binding capacity, a raised cobalamin concentration, an increase in the LDH level, pseudohyperkalemia (an in vitro hyperkalemia secondary to K+ release from platelets), pseudohypoglycemia (secondary to leukocyte utilization in vitro), hypercalcemia, hypergammaglobulinemia, and low leukocyte alkaline phosphatase activity.

  • What is the treatment for CML?

    All the biochemical alterations must be corrected. The hyperuricemia must be treated with adequate hydration and allopurinol. However, the specific treatment for the disease depends on the stage and goal of therapy.

    For chemotherapy, hydroxyurea is used most often because it has fewer side effects than alkylating agents, which can induce aplastic anemia and acute leukemia in patients with CML. Hydroxyurea treatment has a minimal effect on survival, controls the hematologic alterations (without suppressing the Ph1 chromosome), and improves the patient's quality of life.

    P.302


    Both - and -interferons have shown antileukemic activity in the setting of CML; -interferon produces a normalization of blood counts in approximately 75% of patients and suppresses the Ph1 chromosome in approximately 15% of treated patients. The Ph1-negative cell also lacks the bcr rearrangements.

    Drawbacks to interferon treatment are that maintenance therapy is required and it is not free of side effects. Some studies suggest that the prolonged use of interferon (i.e., >1 year) in responders may make patients less responsive to bone marrow transplantation.

    Most recently, tyrosine kinase inhibitors, especially imatinib can lead to a biologic response (normal molecular findings) in more than 50% of patients. These patients may remain in remission for 5 years or more although some patients are starting to show recurrence. Splenic irradiation maybe useful to control splenomegaly and to palliate the symptoms resulting from it. Splenectomy may be useful in carefully selected patients with symptomatic thrombocytopenia, who do not respond to chemotherapy and have a greatly enlarged spleen; however, it is only a palliative measure.

    Allogeneic bone marrow transplantations can be useful in the treatment of some patients with CML. This treatment can eradicate the Ph1-carrying clone and has led to an apparent cure of some patients with CML. However, success with agents such as imatinib and the high toxicity resulting from the procedure, particularly in those who lack suitable donors or are of advanced age, limit its use.

    Leukapheresis can be useful in two types of patients: pregnant women with a very high white blood cell count and hyperleukocytic patients who need rapid cytoreduction to alleviate the signs and symptoms of leukostasis.

  • What is the prognosis in patients with CML?

    In patients with CML who do not attain a cytogenetic response, the median survival ranges from 45 to 60 months. With improved initial therapy approximately 60% to 80% of patients survive 5 years, and 40% survive 8 years.

Case

A 37-year-old white man is seen because of lack of energy, night sweats, and poor appetite with a sensation of fullness after eating even very small amounts of food.

Physical examination reveals signs of anemia, splenomegaly, and the existence of petechiae. A complete blood count is performed and yields the following findings: hematocrit, 25%; platelets, 300,000/mm3, and white blood cells, 72,000/mm3. A bone marrow biopsy is performed and the specimen is found to exhibit a granulocytic erythroid ratio of 10:1 with 100% cellularity and 1% blastocytes.

  • What is the differential diagnosis in this patient, based on the physical examination findings?

  • On the basis of the hematologic findings, what hematopoietic abnormalities would you expect in this patient with suspected CML?

  • What do the bone marrow findings indicate in this patient?

  • P.303


  • What would be the most specific test for establishing the diagnosis of CML in this patient?

  • If the patient is started on single-agent chemotherapy, what would be the likely effect?

Case Discussion

  • What is the differential diagnosis in this patient, based on the physical examination findings?

    When the diagnosis of CML is considered, other possibilities, such as a solid cancer, lymphomas, and chronic infections must be excluded. These other diseases may cause a leukemoid reaction by increased stimulation of normal myelopoiesis. Usually a leukemoid reaction results in a white blood cell count of less than 100,000/mm3, and less than 10% of cells are myelocytes or more immature forms.

    Because normal hematopoiesis is suppressed, the patient could exhibit the signs and symptoms of anemia, such as headache, palpitations, pallor, and cardiac failure. Very rarely, lymph node enlargement is found in patients with CML. Splenomegaly is almost the rule in patients with CML, and it is the source of poor appetite and upper abdominal pain, such as that seen in this patient. Finally, petechiae, although possible, are not very frequent findings in patients with CML.

  • On the basis of the hematologic findings, what hematopoietic abnormalities would you expect in this patient with suspected CML?

    Normal hematopoiesis is suppressed by the leukemic activity in the bone marrow, leading to a decreased number of red blood cells, as well as decreased hemoglobin level and hematocrit. Typically, the anemia of CML is normochromic normocytic. Hypochromic microcytic anemia is typical of iron deficiency.

    Although immature, most of the white blood cells look morphologically normal, and mature neutrophils, band forms, metamyelocytes, and myelocytes constitute most of the white blood cells in this patient. Another characteristic finding is an increased number of basophils. If most of the cells are blasts, this indicates acute leukemia in most cases, although it can also indicate that the patient is in the blastic phase of CML.

  • What do the bone marrow findings indicate in this patient?

    The bone marrow findings are consistent with a diagnosis of CML, and bone marrow biopsy constitutes an important part of the diagnostic evaluation in patients with any kind of leukemia (acute and chronic). Normally, the granulocytic erythroid ratio ranges from 2 to 4:1, but, in the setting of CML, cells of white lineage predominate and increments of any form of white blood cells, from myeloblasts to mature neutrophils, can be found. An increment in lymphocytes and red blood cell precursors is not characteristic of CML. The normal bone marrow cellularity is 50% fat and 50% or less cells, but, in the leukemias, the accelerated production of abnormal cells causes the fat to be replaced, and the cellularity increases to 100%. Finally, even in normal bone marrow, a very small number of blast cells can be found; in CML, a small percentage of blast cells can be found, but this does not

    P.304


    necessarily signify acute leukemia. In blast crisis or acute leukemia, at least 20% of the cells in the bone marrow are blast cells.

  • What would be the most specific test for establishing the diagnosis of CML in this patient?

    The most specific test for establishing the diagnosis of CML is a cytogenetic investigation for the Ph1 chromosome, or t(9;22), which is found in 90% of cases of CML. Of the remaining 10% at least half will have bcr rearrangements measured by in situ by hybridization.

  • If the patient is started on single-agent chemotherapy, what would be the likely effect?

    The chemotherapeutic agent most commonly used in the treatment of CML is hydroxyurea. This therapy can improve the patient's quality of life by rapidly decreasing the number of white blood cells and platelets. It does not prolong survival very much, if at all, in patients with CML. The interferons can induce complete hematologic and cytogenetic remissions, with suppression of the Ph1 chromosome in patients with CML. Most importantly tyrosine kinase inhibitors have high incidence of biologic responses and less toxicity.

    Allogeneic bone marrow transplantation has been the only curative treatment for CML but has a high rate of complications. Advanced age and the lack of suitable donors preclude its use in many patients, but it may be the therapy of choice in this 37-year-old man if he does not attain a biologic remission or relapses after this remission is attained.

Suggested Readings

Canellos G. Clinical characteristics of the blast phase of chronic myelogenous leukemia. Hematol Oncol Clin North Am 1990;4:359.

Kurzrock R, Gutterman JU, Talpaz M. The molecular genetics of Philadelphia chromosome positive leukemias. N Engl J Med 1988;319:990.

Quintas-Cardama A, Cortes JE. Chronic myeloid leukemia: diagnosis and treatment. Mayo Clin Proc 2006;81:973.

Reiter E, Greinix HT, Brugger S, et al. Long term follow up after allogeneic stem cell transplantation for chronic myelogenous leukemia. Bone Marrow Transplant 1998;4:S86.

Rodriguez J, Cortes J, Smith T, et al. Determinations of prognosis in late chronic-phase chronic myelogenous leukemia. J Clin Oncol 1998;16:3782.

Colon Cancer

  • What is the incidence of colon cancer?

  • What are some of the known risk factors for colon cancer?

  • Should patients be screened for colon cancer?

  • What is the current treatment for primary colon cancer?

  • What staging procedures need to be done to adequately stage a patient with colon cancer?

  • What is the staging system for colon cancer?

  • P.305


  • What is the prognosis for patients with colon cancer, based on their stage?

  • What should the follow-up consist of in patients with colon cancer after they have undergone primary surgical resection for curative intent?

  • Are there any effective adjuvant treatments to decrease the risk of recurrence in patients with colon cancer who have undergone resection?

  • Is there any effective chemotherapy for patients with metastatic disease?

Discussion

  • What is the incidence of colon cancer?

    There are more than 140,000 new cases of colon cancer each year in the United States. It affects approximately 1 of every 20 people in Western cultures and accounts for 15% of all cancers. In the United States, the actual incidence rate is approximately 35 cases per 100,000 population per year.

  • What are some of the known risk factors for colon cancer?

    There are several inherited colonic polyposis syndromes associated with an increased risk of cancer of the large bowel. The most important one is the familial adenomatosis syndrome, which is inherited as an autosomal dominant trait. In affected people, polyps develop over the entire length of the colon by 30 years of age. If a total colectomy is not performed, the cancer rate escalates to as high as 80% to 90% by 45 years of age. There are also other, less-frequent polyposis syndromes predisposing to colon cancer.

    There appears to be a certain genetic tendency toward colon carcinoma that is independent of the inherited polyposis syndromes. First-degree relatives of people with colon cancer diagnosed before the age of 60 have a two- to threefold greater chance of acquiring colon cancer than the general population.

    Patients with inflammatory bowel disease are also at increased risk for colon cancer. Those with ulcerative colitis have approximately a 50% to 60% chance for development of large bowel carcinoma if a colectomy is not performed. Crohn's disease is also associated with an increased risk of colon cancer, but to a much lesser degree than ulcerative colitis.

    The findings from several population studies have suggested that diet plays a large role in the development of colon cancer. Cultures in which the populace consumes a high-fat, low-fiber diet exhibit an increased incidence of colon cancer, compared with cultures in which a low-fat, high-fiber diet is consumed. Daily aspirin may help prevent colon cancer.

  • Should patients be screened for colon cancer?

    The prognosis for colon cancer is dramatically improved the earlier it is detected and treated. Screening programs are aimed at detecting colon cancers at an early stage and have led to an improvement in survival and in the risk of relapse. Most screening programs are usually directed at populations with a high risk for colon cancer, including the groups already mentioned.

    Screening techniques for colon cancer comprise digital rectal examination, the testing of stool for occult blood, sigmoidoscopy with an air-contrast barium enema, and colonoscopy. Recommendations are that people should be checked

    P.306


    for occult blood at 50 years of age and yearly thereafter, with colonoscopy also performed at 50 years and every 5 to 10 years, if negative thereafter.

  • What is the current treatment for primary colon cancer?

    The primary treatment for colon cancer is surgical. Once the cancer has been diagnosed and preoperative staging performed, the patient should be referred to an oncologic surgeon for definitive treatment. The exact surgical approach used is dictated by the tumor's location in the colon. For true colon cancers (i.e., cancers above the peritoneal reflection), a hemicolectomy is usually performed. For rectal carcinomas (i.e., tumors below the peritoneal reflection), a low anterior resection or an abdominoperineal resection is performed. Regardless of the surgical procedure, a thorough exploration of the entire abdomen, including the liver, should be carried out and any suspect lesions sampled for biopsy.

  • What staging procedures need to be done to adequately stage a patient with colon cancer?

    The preoperative staging evaluation of patients with colon cancer includes history taking, physical examination, complete blood count, liver function tests, the carcinoembryonic antigen (CEA) level, and a chest radiograph. Before surgery further investigation by computed tomographic (CT) scanning or positron-emission tomography (PET) should be performed. Surgical and pathologic staging should then be done to determine the exact stage of the disease. If the preoperative CEA level is elevated, repeat measurement should be performed approximately 1 month after surgery to see if it returns to normal.

  • What is the staging system for colon cancer?

    There are many staging systems for colon cancer. The most widely used is the Aster-Coller modification of the Dukes' staging system (Table 7-7). It is based on the depth of tumor invasion, regional lymph node involvement, and distant metastasis.

    Table 7-7 The TNM Staging Modi.cation of the Dukes' Staging System

    Stage Depth of Invasion Lymph Node
    Status
    Metastases
    Distant
    Stage I Invades submucosa Negative Absent
    Stage I Invades muscularis propria Negative Absent
    Stage II Invades serosa Negative Absent
    Stage IIB Invades through bowel wall into adjacent organs Negative Absent
    Stage IIIA Invades positive muscularis propria Positive Absent
    Stage IIIB Invades positive serosa Positive Absent
    Stage IIIC Invades through bowel wall into adjacent organs Positive Absent
    Stage IV Any depth of invasion Present

    P.307


    Table 7-8 Five-Year Survival Rates for Aster-Coller Stages

    Stagea 5-y Survival (%)
    Stage I 90 95
    Stage IIA 78
    Stage IIB 63
    Stage IIIA 74
    Stage IIIB 48
    Stage IIIC 38
    Stage IV <5
    aSee Table 7-7 for definition of stages.

  • What is the prognosis for patients with colon cancer, based on their stage?

    Table 7-8 lists the 5-year survival rates for the various stages of the TNM staging IV and Aster-Coller modification of the Dukes' staging system; this takes into account improvement in survival with adjuvant chemotherapeutic regimens.

  • What should the follow-up consist of in patients with colon cancer after they have undergone primary surgical resection for curative intent?

    Routine scheduled follow-up is very important for the early detection of local and distant recurrences, as well as new primary colon cancer. Patients should be seen every 3 to 4 months for the first 3 years. Follow-up evaluation should include history, physical examination, liver function tests, measurement of the CEA level, and complete blood counts. Colonoscopy and a chest radiograph should be obtained yearly. CT scanning should be performed for further evaluation of rising liver function tests or CEA levels. After 3 years, the interval between these evaluations can be increased.

  • Are there any effective adjuvant treatments to decrease the risk of recurrence in patients with colon cancer who have undergone resection?

    Several studies have shown that 6 months of postoperative treatment with 5-fluorouracil, leucovorin, and oxaliplatin can decrease by 50% the likelihood of stage III colon cancer recurrence. The death rate in this setting was reduced by 33%. More recently, selected studies of 5-fluorouracil, leucovorin, and oxaliplatin therapy have shown a decrease in both recurrence and death rates for patients with late stage IIB colon cancer.

  • Is there any effective chemotherapy for patients with metastatic disease?

    5-Fluorouracil, leucovorin, and oxaliplatin therapy (which modulates oxaliplatin and bevacizumab angiogenesis) has led to an increase in the response rates in metastatic disease from 60% to 80%. The use of biologic agents such as antiangiogenesis factors alone and arbitux (anti-EGFR) increased survival in stage IV patients.

Case

A 72-year-old white man is seen in the emergency room because of severe fatigue and vague abdominal discomfort. He has no significant past medical history other than slight

P.308


anemia, which was noted during a physical examination 3 years ago. His hematocrit at that time was 37%, and white blood cell and platelet counts were normal. Physical examination was remarkable only for cachexia and a pale appearance. His initial laboratory values at the current time are as follows: white blood cell count, 7,800/mm3; hemoglobin, 7 g/dL; hematocrit, 21%; platelets, 600,000/mm3; MCV, 62 fL; AST, 89 mU/mL (normal, 0 to 35 mU/mL); ALT, 129 mU/mL (normal, 0 to 38 mU/mL); alkaline phosphatase, 360 mU/mL (normal, 0 to 125 mU/mL); and total bilirubin, 0.7 mg/dL (normal, <1.0 mg/dL).

The patient is admitted to the hospital for blood transfusion and evaluation of his anemia. The admission chest radiograph shows numerous pulmonary nodules, and a barium enema examination reveals a near-obstructing lesion at the hepatic flexure. A CT scan of the liver depicts numerous low-density lesions in both lobes of the liver. Colonoscopy is performed, and this reveals a mucosal lesion at the hepatic flexure. Biopsy of this lesion reveals adenocarcinoma.

  • What is the cause of this patient's anemia?

  • Would earlier diagnosis of the cause of this patient's anemia have made any difference?

  • What type of treatment would you now advise for the lesion in the hepatic flexure of the colon?

  • Would you recommend any other treatments for the lesions in the lung or liver?

  • What stage is this patient's cancer?

  • What is the prognosis in this patient?

Case Discussion

  • What is the cause of this patient's anemia?

    The patient almost certainly has iron-deficiency anemia secondary to the chronic blood loss in the stool stemming from a bleeding colon cancer.

  • Would earlier diagnosis of the cause of this patient's anemia have made any difference?

    It is difficult to say whether an earlier diagnosis would have definitely made a difference. It should certainly have been possible to diagnose the colon cancer, and at an earlier stage the prognosis would likely have been better. Recommended screening procedures, particularly colonoscopy, would probably have improved this patient's outcome.

  • What type of treatment would you now advise for the lesion in the hepatic flexure of the colon?

    The patient needs to undergo a hemicolectomy to prevent obstruction. This is only for palliation and will not affect the overall prognosis.

  • Would you recommend any other treatments for the lesions in the lung or liver?

    This patient has metastatic disease and should be offered systemic chemotherapy consisting of 5-fluorouracil, oxaliplatin, and leucovorin with an antiangiogenesis factor: the standard approach for metastatic colon cancer. The best doses and schedules of administration are yet to be determined. If the patient does not respond to this standard therapy he should be enrolled in a clinical trial.

  • P.309


  • What stage is this patient's cancer?

    This patient clearly has metastatic disease, and therefore is in stage D according to the Aster-Coller modification of the Dukes' staging system.

  • What is the prognosis in this patient?

    This patient has incurable cancer. He has approximately a 30% to 45% chance of responding to standard therapy. If he responds, he will likely live longer. He has approximately a 35% to 40% chance of 5-year survival.

Suggested Readings

Midgley R, Kerr D. Colorectal cancer. Lancet 1999;353:391.

Smith RE, Colangelo L, Wieand HS, et al. Randomized trial of adjuvant therapy in colon carcinoma: 10-year results of NSABP protocol C-01. J Natl Cancer Inst 2004;96:1128.

Steele G Jr. Combined-modality therapy for rectal carcinoma: the time has come. N Engl J Med 1991;324:764.

Steele G Jr, Bleday R, Mayer RJ, et al. A prospective evaluation of hepatic resection for colorectal carcinoma metastases to the liver: Gastrointestinal Tumor Study Group Protocol 6584. J Clin Oncol 1991;9:1105.

Steele G Jr, Burt R, Winawer SJ, eds. Basic and clinical perspectives of colorectal polyps and cancer. New York: Alan R. Liss, 1988.

Erythrocytosis

  • What is erythrocytosis?

  • What are the two major types of erythrocytosis?

  • What are some causes of secondary erythrocytosis due to appropriate erythropoietin secretion?

  • What are some causes of secondary erythrocytosis due to inappropriate erythropoietin secretion?

  • What is polycythemia vera?

  • What are the symptoms of polycythemia vera?

  • How is polycythemia vera diagnosed?

  • What is the likely length of survival in a patient with polycythemia vera?

  • What is the rare hepatic complication that can arise in patients with polycythemia vera?

Discussion

  • What is erythrocytosis?

    A patient with a hematocrit greater than 55% that is not due to dehydration is considered to have an erythrocytosis. The chromium 151 labeled red blood cell measurement of the total red blood cell mass is, however, the gold standard for establishing the diagnosis, but if the patient has a hematocrit greater than 60%, no further studies are necessary.

  • P.310


  • What are the two major types of erythrocytosis?

    When the elevated hematocrit is due to increased erythropoietin secretion, this constitutes secondary erythrocytosis. Primary erythrocytosis is caused by increased red blood cell production that does not stem from increased erythropoietin secretion.

  • What are some causes of secondary erythrocytosis due to appropriate erythropoietin secretion?

    Any disorder that causes tissue hypoxia stimulates the renal production of erythropoietin. These disorders include chronic obstructive lung disease, living at high altitudes, hemoglobin (Hb Chesapeake and methemoglobin) that does not release oxygen correctly, or cardiac disease that causes right-to-left shunting. In relative erythrocytosis, the red blood cell mass is normal, and this occurs in the settings of dehydration or decreased plasma volume.

  • What are some causes of secondary erythrocytosis due to inappropriate erythropoietin secretion?

    Many disease states can be associated with increased erythropoietin production. Diseased kidneys may secrete erythropoietin inappropriately or tumors may secrete hormones that function like erythropoietin. Renal, adrenal, or hepatic tumors, ovarian carcinoma, or benign uterine myomas all secrete erythropoietin-like substances. Other causes of increased erythropoietin secretion are renal artery stenosis, hydronephrosis, renal cysts, or renal transplantation.

  • What is polycythemia vera?

    Polycythemia vera is an absolute erythrocytosis secondary to the clonal expansion of red blood cells, making it a myeloproliferative disorder.

  • What are the symptoms of polycythemia vera?

    Symptoms stem from vascular congestion or obstruction due to increased blood viscosity. Patients complain of headaches, itching and burning feet, or malaise. The retinal veins become engorged and hepatosplenomegaly may be present. The incidence of cardiovascular and cerebrovascular disease is increased in these patients because of the elevated blood viscosity.

  • How is polycythemia vera diagnosed?

    An increased red blood cell mass, an oxygen saturation of 92% or more, and splenomegaly are the cardinal signs of polycythemia vera. If splenomegaly is not present, polycythemia vera is evident if the patient has a platelet count that exceeds 400,000/mm3, a white blood cell count greater than 12,000/mm3, a cobalamin level of 900 pg/mL or greater, or an elevated neutrophil alkaline phosphatase score.

  • What is the likely length of survival in a patient with polycythemia vera?

    Without treatment, half the patients die within 24 months, usually due to vascular disease. Phlebotomy to maintain a hematocrit of 45% can prolong the life span to more than 6 years, and median survival in patients who receive effective chemotherapy is 12.5 years.

  • P.311


  • What is the rare hepatic complication that can arise in patients with polycythemia vera?

    Budd-Chiari syndrome is an occlusion of the hepatic veins sometimes seen in patients with polycythemia vera and other diseases that increase blood viscosity. This causes right upper quadrant pain and elevations in the liver enzyme levels. It is very difficult to treat, and is best prevented by treating the erythrocytosis aggressively.

Case

A 55-year-old man who is a smoker and has hypertension sees his internist because of malaise and nasal stuffiness with full sensation in his frontal sinuses. On further questioning, the patient also describes having itchy, red feet that worsen in the shower. The patient has no shortness of breath with activity and does not snore or experience daytime drowsiness.

Physical examination reveals a plethoric patient who is in no acute distress. His lungs are clear to auscultation. His liver span is 18 cm and his spleen tip is palpable.

The following laboratory values are reported: hematocrit, 65%; white blood cell count, 8,500/mm3; platelets, 210,000/mm3; and differential: 50% segmented neutrophils, 30% lymphocytes, 3% basophils, and 10% monocytes.

Arterial blood gas determinations performed on room air reveal a partial pressure of oxygen of 65 mm Hg, a partial pressure of carbon dioxide of 38 mm Hg, and an oxygen saturation of 93%.

  • What is the diagnosis in this patient?

  • Why is it important to know whether the patient snores or experiences daytime drowsiness?

  • What is the cause of this patient's nasal stuffiness?

  • What should be the initial treatment in this patient?

  • What is this patient's prognosis?

Case Discussion

  • What is the diagnosis in this patient?

    This patient most likely has polycythemia vera. The oxygen saturation greater than 90% and the presence of splenomegaly support the diagnosis. The presence of mononuclear and basophilic cells also supports the diagnosis of a myeloproliferative disorder, which would be further supported by a bone marrow biopsy that shows trilinear hyperplasia.

  • Why is it important to know whether the patient snores or experiences daytime drowsiness?

    Snoring and daytime drowsiness are symptoms of sleep apnea, a cause of secondary erythrocytosis. Although phlebotomy can cure the patient's erythrocytosis,

    P.312


    it cannot treat the nighttime hypoxia or sleep apnea, and the patient could go on to have right-sided heart failure.

  • What is the cause of this patient's nasal stuffiness?

    Although he may have a sinus infection, the nasal stuffiness is most likely due to increased blood viscosity.

  • What should be the initial treatment in this patient?

    Phlebotomy should be performed as soon as possible to decrease the hematocrit to 45% to 50%. The increased blood viscosity places this patient who has two other risk factors for atherosclerotic disease, namely smoking and hypertension, at risk for a stroke or cardiovascular accident.

  • What is this patient's prognosis?

    Even with careful treatment of his erythrocytosis with phlebotomy and chemotherapy, his life expectancy will probably be more limited because of his smoking and hypertension.

Suggested Readings

Conley CL. Polycythemia vera, diagnosis and treatment. Hosp Pract 1987;22:107.

Ellis JT, Peterson P, Geller SA, et al. Studies of the bone marrow in polycythemia vera and the evolution of myelofibrosis and second hematologic malignancies. Semin Hematol 1986;23:144.

Murphy S. Diagnostic criteria and prognosis in polycythemia vera and essential thrombocytopenia. Semin Hematol 1999;36:9.

Schwarts RS. Polycythemia vera: chance, death, and mutability [Editorial]. N Engl J Med 1998;338:613.

Lymphomas

  • How and when does Hodgkin's disease typically present?

  • What is the relationship between the histologic patterns and the stage in Hodgkin's disease?

  • Of what should the staging evaluation in patients with Hodgkin's disease consist, and how do the findings have an impact on therapy?

  • What are the cure rates and the long-term sequelae of the treatment for Hodgkin's disease?

  • What are the known causes or diseases associated with the development of non Hodgkin's lymphoma?

  • How does the World Health Organization's working formulation of non Hodgkin's lymphoma differ from the older classifications?

  • How does the biology of high-grade lymphoma differ from that of low-grade lymphoma, and how does this affect treatment and survival?

  • What type of lymphoma typically involves the skin, and how does this influence staging and treatment?

  • P.313


  • What population of patients is prone to acquiring secondary CNS lymphoma, and can this be prevented?

  • What complication of therapy can occur in the setting of rapidly growing tumors, and how can this be prevented?

Discussion

  • How and when does Hodgkin's disease typically present?

    Hodgkin's disease typically presents in adolescence or young adulthood. However, a bimodal age distribution has been observed, especially in developed countries. The first peak is in adolescence or young adulthood, whereas the second peak occurs at 55 years of age. Hodgkin's disease typically presents as a waxing and waning adenopathy, most commonly in the neck or supraclavicular area. Fifty percent of patients present with a mediastinal mass visible on chest radiography, and 40% present with B symptoms (fever, night sweats, and 10% weight loss in the preceding 6 months).

  • What is the relationship between the histologic patterns and the stage in Hodgkin's disease?

    There are four distinct histologic patterns seen in Hodgkin's disease, all of which possess the Sternberg-Reed cell. The four histologic patterns and their prevalences are nodular sclerosis (70%), lymphocyte predominance (15%), mixed cellularity (10%), and lymphocyte depletion (5%).

    Hodgkin's disease is staged according to the Ann Arbor classification:

    • Stage I. Involvement of a single lymph node region (I) or a single extralymphatic organ or site (IE).

    • Stage II. Involvement of two or more lymph node regions on the same side of the diaphragm (II), or localized involvement of two or more extralymphatic organs or sites (IIE).

    • Stage III. Involvement of lymph node regions on both sides of the diaphragm (III), or localized involvement of an extralymphatic organ or site (IIIE) or spleen (IIIS), or both (IIISE). III1 refers to involvement of lymph nodes in the upper abdomen; III2 refers to involvement of lower abdominal nodes.

    • Stage IV. Diffuse or disseminated involvement of one or more extralymphatic organs, with or without associated lymph node involvement. The organ, or organs, involved may be identified by a symbol.

    • Also: A = asymptomatic; B = fever, night sweats, and weight loss exceeding 10% of the total body weight.

    Hodgkin's disease spreads through contiguous lymph nodes; however, the spleen is commonly the only site of involvement in the abdomen, and its involvement is thought to be due to lymphatic spread.

    Histologic progression in Hodgkin's disease involves the progressive loss of lymphocytes. For example, lymphocyte predominance can progress to mixed cellularity and eventually lymphocyte depletion. Those patients who present with nodular sclerosis may also experience some changes in histologic type, although most do not show obvious histologic progression. The histologic

    P.314


    pattern also correlates with the stage of the disease, and thereby the prognosis. Nodular sclerosis and lymphocyte predominance are more commonly seen in early disease (stages I and II). Therefore, these histologic patterns are associated with a better outcome. Mixed cellularity and lymphocyte depletion are associated with a poorer prognosis and are often seen in patients with advanced disease.

  • Of what should the staging evaluation in patients with Hodgkin's disease consist, and how do the findings have an impact on therapy?

    The staging evaluation in patients with Hodgkin's disease includes a complete history and physical examination. Laboratory investigations should include a complete blood count and evaluation of the smear for changes indicating anemia, hemolysis, or abnormal white blood cells as well as a differential, determinations of the sedimentation rate and alkaline phosphatase level, and evaluation of liver and renal function. The radiologic evaluation should always include CT studies of the chest, abdomen, and pelvis or CT/PET scans.

    A diagnosis based on tissue findings is a must. Needle aspiration or cytologic findings is not adequate because the tissue obtained by these methods yields no information about the nodal architecture. It is preferable to obtain a lymph node or wedge of a large mass, but even then it may take more than one lymph node biopsy to document the presence of the disease if only reactive hyperplasia is seen. Bone marrow biopsy is a required part of the staging workup, particularly in symptomatic patients, but should not be substituted for the tissue examination because, again, the nodal architecture cannot be observed. General guidelines no longer suggest performing a staging laparotomy unless if the results would affect the nature of therapy. This may happen in early stage disease (i.e., stages IB, IIB, and IIIA), when the findings from laparotomy could alter a decision to use radiation therapy alone.

    The choice of therapy in patients with Hodgkin's disease is governed by stage. Patients with stage I and II disease can be treated with radiation therapy alone. If there is bulky disease, combined chemotherapy and irradiation should be used. For patients with stage III and IV disease, chemotherapy should be used with radiation delivered to sites of bulky disease. There is still some controversy about what is the best treatment for stage IIB and IIIA disease. Most therapeutic options have high remission rates, and frequently long-term side effects dictate the choice.

  • What are the cure rates and the long-term sequelae of the treatment for Hodgkin's disease?

    Combination chemotherapy and advances in radiation therapy have achieved overall cure rates of approximately 70% in patients with Hodgkin's disease. The cure rates seen for early-stage disease are greater, with a 90% to 95% long-term survival rate observed for patients with stage I and II disease. The great number of survivors has allowed long-term follow-up and a study of the effects of combination chemotherapy and radiation therapy.

    Hodgkin's disease is associated with immunologic abnormalities involving changes in both lymphocyte function and humoral immunity. These defects

    P.315


    are aggravated by treatment and this increases the risk of such infections as disseminated herpes zoster. This immune dysfunction can last for years after treatment. Treatment with chemotherapy is associated with an increased risk of secondary leukemia, which is further increased in patients older than 40 years, heavily treated patients (both chemotherapy and radiation therapy), and those who undergo prolonged therapy with alkylating agents.

    The sequelae of the therapy for Hodgkin's disease are various. It is important to be aware of them, but it is also important that therapy not be severely modified (i.e., a lower dosage of either chemotherapy or radiation therapy) to minimize risk, because attempting to minimize the risk in this manner may compromise cure.

  • What are the known causes or diseases associated with the development of non Hodgkin's lymphoma?

    The risk of lymphoma is increased in patients with certain connective tissue and immunologic disorders. These include human immunodeficiency virus (HIV) infection, Klinefelter's syndrome, acquired hypogammaglobulinemia, iatrogenic immunosuppression (especially after organ transplantation), ataxia telangiectasia syndrome, Sj gren's syndrome, rheumatoid arthritis and systemic lupus erythematosus, Swiss-type agammaglobulinemia, common variable immunodeficiency disease, acquired immunodeficiency syndrome, and the X-linked lymphoproliferative syndrome.

    A viral etiology of lymphoma has been proposed, but no clear proof of this virus exists except for human T-cell leukemia virus type 1 (HTLV-1) infection. Certain types of more common lymphomas have been associated with a viral etiology (e.g., Burkitt's lymphoma and Epstein-Barr virus and HIV or post organ transplantation lymphomas). The search to establish a viral cause has implicated oncogenes, leading to the identification of various cytogenetic abnormalities in lymphoma. The common pattern is for a known oncogene to be translocated into an immunoglobulin gene locus. The common translocations are listed in Table 7-9.

  • How does the World Health Organization's working formulation of non Hodgkin's lymphoma differ from the older classifications?

    The non Hodgkin's lymphomas are classified according to histologic type. The newer World Health Organization working formulation is based on the morphologic features of each type of lymphoma. This classification divides

    P.316


    the lymphomas into three major subgroups: low grade, intermediate grade, and high grade. Therefore, the lymphomas are classified according to both their morphologic features and their behavior. Newer classifications examine molecular characteristics, as shown in Table 7-10.

    Table 7-9 Common Translocations in Patients with Lymphoma

    Translocation Histologic Type of Lymphoma
    t(8;14) chromosome Burkitt's and non-Burkitt's
    t(2;8) chromosome Burkitt's and non-Burkitt's
    t(8;22) chromosome Burkitt's and non-Burkitt's
    t(14;18) chromosome Follicular
    t(2;5) chromosome Anaplastic large cell

  • How does the biology of high-grade lymphoma differ from that of low-grade lymphoma, and how does this affect treatment and survival?

    High-grade non Hodgkin's lymphoma is a group of diseases that behave aggressively, especially compared with the behavior that is typical of low-grade

    P.317


    non Hodgkin's lymphoma or Hodgkin's disease. The mean survival in patients with high-grade disease who do not respond to therapy is 2 years, whereas patients with low-grade disease can live for up to 20 years.

    Table 7-10 WHO Classi.cation of the Non Hodgkin's Lymphomas

    • The indolent lymphomas
    • B-cell neoplasms
      • Small lymphocytic lymphoma/B-cell chronic lymphocytic leukemia
      • Lymphoplasmacytic lymphoma (Waldenstrom's macroglobulinemia)
      • Plasma cell leukemia
      • Hairy cell leukemia
      • Follicular lymphoma (grade 1 and 2)
      • Marginal cell lymphomaa
    • T-cell neoplasms
      • T-cell large granular lymphocyte leukemia
      • Mycosis fungoides
      • T-cell prolymphocytic leukemia
    • Natural killer cell neoplasms
      • Natural killer cell large granular lymphocyte leukemia
    • The aggressive lymphomas
    • B-cell neoplasms
      • Follicular lymphoma (grade 3)
      • Diffuse large B-cell lymphoma
      • Mantle cell lymphomaa
    • T-cell neoplasm
      • Peripheral T-cell lymphoma
      • Anaplastic large cell lymphoma, T/null cell
    • The highly aggressive lymphomas
    • B-cell neoplasms
      • Burkitt's lymphoma
      • Precursor B-lymphoblastic leukemia/lymphoma
    • T-cell neoplasms
      • Adult T-cell lymphoma/leukemia
      • Precursor T-lymphoblastic leukemia/lymphoma
    aMarginal or mantle cell lymphoma can behave clinically as either indolent or an aggressive disorder.
    WHO, World Health Organization.
    Adapted from Harris NL, Jaffe ES, Diebold J, et al. World Health Organization classification of neoplastic diseases of the hematopoietic and lymphoid tissues: report of the Clinical Advisory Committee. Airlie House, Virginia: 1997; J Clin Oncol 1999;17:3835.

    Patients with high-grade lymphoma can present with localized disease (<20%), but more commonly are in an advanced stage. There can be involvement of either extranodal (35%) or privileged sites (the CNS or testes). Most disease is of B-cell origin (85%), with the remainder of T-cell origin.

    Poor prognostic factors include poor performance status, bulky disease (>10 cm), high LDH level (>500 IU/dL), bone marrow involvement, and B symptoms.

    High-grade non Hodgkin's lymphomas are very sensitive to chemotherapy, and aggressive treatment is the only chance for cure. Up to 60% of patients can be cured with the newer chemotherapeutic regimens. This is in sharp contrast to the experience with low-grade lymphomas, in which cure rates of less than 20% are seen and survival does not seem to be affected by the type of response to chemotherapy.

  • What type of lymphoma typically involves the skin, and how does this influence staging and treatment?

    Lymphomatous involvement of the skin is commonly seen in the setting of T-cell lymphoma. It occurs in approximately 10% of all cases of non Hodgkin's lymphoma, and this group of diseases is called cutaneous T-cell lymphoma (CTCL). The low-grade form of CTCL is mycosis fungoides or Sezary syndrome. Sezary syndrome is diagnosed in the setting of mycosis fungoides when the malignant cells (Sezary cells) are found in the peripheral blood.

    The staging classification for CTCL differs from that for other forms of non Hodgkin's lymphoma, and is based on the TNM system, as shown in Table 7-11.

    The mainstay of management of early mycosis fungoides and Sezary syndrome has been topical treatment, but there is little evidence that this prolongs survival.

  • What population of patients is prone to acquiring secondary CNS lymphoma, and can this be prevented?

    CNS involvement is rarely seen in patients with low-grade lymphoma. When seen, a histologic transformation to high-grade lymphoma should be suspected. Within this group, CNS involvement is more common when Waldeyer's tonsillar ring, the bone marrow, or the testes are affected. Among the high-grade lymphomas, there is a group of especially aggressive lymphomas, and these consist of undifferentiated lymphomas (Burkitt's and non Burkitt's type), lymphoblastic lymphoma, and acute T-cell lymphoma. The incidence of CNS involvement is high in these patients and, therefore the CNS should be treated prophylactically with intrathecal chemotherapy. Patients on immunosuppression after organ transplantation [posttransplant lymphoproliferative disease (PTLD)] often present with CNS lymphoma.

    P.318


    Table 7-11 National Cutaneous T-Cell Lymphoma Workshop Staging Classificationa

    Tumor (T) Skin Node (N) Lymph Nodes Metastasis (M) Visceral Organs
    T1 Limited plaques (<10% body surface area) N0 No adenopathy, histology negative M0 No involvement
    T2 Generalized plaques N1 Adenopathy; histology negative M1  
    T3 Cutaneous tumors N2 No adenopathy; histology positive    
    T4 Generalized erythroderma N3 Adenopathy; histology positive    
    Stage I: Limited (IA) or generalized (IB) plaques without adenopathy or histologic involvement of lymph nodes or viscera (T1 N0 M0 or T2 N0 M0)
    Stage II: Limited or generalized plaques with adenopathy (IIA) or cutaneous tumors with or without adenopathy (IIB); without histologic involvement of lymph nodes or viscera (T1 2 N1 M0 or T2 N0 1 M0)
    Stage III: Generalized erythroderma with or without adenopathy; without histologic involvement of lymph nodes or viscera (T4 N0 2 M0)
    Stage IV: Histologic involvement of lymph nodes (IVA) or viscera (IVB) with any skin lesion and with or without adenopathy (T1 4 N2 3 M0 for IVA; T1 4 N0 3 M1 for IVB)
    aBlood involvement should be recorded as absent (B0) or present (B1) but is not currently used to determine final stage.

  • What complication of therapy can occur in the setting of rapidly growing tumors, and how can this be prevented?

    The tumor lysis syndrome can occur in the setting of tumors that are exquisitely sensitive to chemotherapy and is seen when there is a large tumor burden. The syndrome is characterized by hyperuricacidemia, hyperphosphatemia, hyperkalemia, and hypocalcemia, and can result in acute renal failure and sudden death, if not treated. Fortunately, if the signs are carefully watched for, the patient can be spared its effects. The management of this syndrome includes aggressive hydration, the alkalinization of urine, and allopurinol therapy before and during chemotherapy.

Case

A 42-year-old woman is referred to you by her family physician for the evaluation of bilateral neck adenopathy. She has noticed this swelling intermittently for approximately 6 months. She has occasionally noticed axillary node swelling but denies any other adenopathy. She has noticed that she tires more easily and seems to pick up every little virus. She admits to experiencing occasional early satiety, but denies any increase in abdominal girth or changes in bowel habits. She denies any fever, chills, night sweats, weight loss, or change in appetite.

Her family history is remarkable for a mother with breast cancer (the patient's last mammogram 1 year ago was normal). She does not smoke or drink.

P.319


Physical examination findings are remarkable for bilateral neck and axillary adenopathy. She has no oral or pharyngeal lesions and no breast masses. Her spleen is mildly enlarged but her liver size is normal. She has no other physical abnormalities.

Laboratory findings are remarkable for a mild normochromic, normocytic anemia (hemoglobin, 13.0 g/dL; hematocrit, 39%); the platelet count is 250,000/mm3 and the white blood cell count is 5,200/mm3 with a normal differential. A chemistry panel is remarkable for a slightly elevated LDH level, but the AST, ALT, bilirubin, and alkaline phosphatase values are normal. Her chest radiographic study is normal.

A staging evaluation is done and reveals the following findings. Tissue analysis reveals malignant lymphoma consisting of follicular small cleaved (nodular poorly differentiated) cells that are CD20 positive. Bone marrow biopsy reveals normal cellularity with lymphoid follicles (normal for age), a slight increase in the number of erythroid precursors, normal megakaryocytes, and a decrease in the iron content. Cytogenetic examination identifies a balanced translocation, t(14;18). CT scan of the abdomen depicts moderate splenomegaly and mild retroperitoneal adenopathy. Serum immunoelectrophoresis reveals mild hypogammaglobulinemia with a monoclonal immunoglobulin M (IgM) spike.

  • On the basis of the physical examination and laboratory findings, what is the differential diagnosis in this patient?

  • On the basis of the findings from the staging evaluation, what stage of non Hodgkin's lymphoma is this patient in, and what are her treatment options and prognosis?

  • What are the implications of her cytogenetic abnormalities?

  • Is it further necessary to evaluate or treat her hypogammaglobulinemia?

  • What is the significance of the monoclonal IgM spike in this patient?

    The patient is observed to do fine at her 6-month visits, until 4 years later, when painful and enlarging nodes develop.

  • What form of therapy would you offer her when the painful and enlarging nodes are detected, and what outcome can she expect?

    With the onset of therapy consisting of oral alkylating agents, a severe anemia develops in this patient, requiring transfusion.

  • How would you evaluate the anemia that develops with the alkylating agent therapy?

    The results of investigations performed to determine the source of her anemia are as follows: Coombs' direct and indirect test, positive; reticulocyte count, 9%; blood smear, spherocytes and increased reticulocytes; and bone marrow biopsy, increased cellularity with erythroid hyperplasia plus the presence of small lymphocytes, suggesting lymphomatous involvement.

  • On the basis of the findings yielded by the investigations for her anemia, what are the treatment options at this point?

    This patient does well for 6 months with monthly intravenous chemotherapy and immunoglobulin therapy, as needed. Two years after the start of therapy, increased splenomegaly develops that appears refractory to the previous chemotherapy.

  • With the appearance of increased splenomegaly, what is the differential diagnosis, and how should you confirm it?

  • What are the treatment options in this patient whose disease is now in an advanced stage?

P.320


Case Discussion

  • On the basis of the physical examination and laboratory findings, what is the differential diagnosis in this patient?

    The neck adenopathy in this patient can represent a normal finding; 50% of patients can have lymph nodes that are less than 0.5 cm in diameter. It can also signify acute infection stemming from acute viral infections, mononucleosis, toxoplasmosis, or pulmonary infections, but in this setting the nodes are usually firm and tender and recede within 2 to 4 weeks. Solid tumors are also a consideration in the differential diagnosis, and include head and neck cancer, as well as thymic, lung, and breast cancer; lung and breast cancers are more commonly associated with supraclavicular and axillary adenopathy. A fourth possibility is Hodgkin's disease or non Hodgkin's lymphoma. Patients with lymphoma can have lymph nodes that come and go.

  • On the basis of the findings from the staging evaluation, what stage of non Hodgkin's lymphoma is this patient in, and what are her treatment options and prognosis?

    Non-Hodgkin's lymphoma is usually staged according to the Ann Arbor classification used for Hodgkin's disease. According to this system's criteria, this patient has stage III disease. In the setting of low-grade lymphoma, it is important to identify localized versus disseminated disease. However, prognostic factors are often most important. This patient has disease above and below the diaphragm as well as probable splenic involvement. She does not clearly have bone marrow involvement, because lymphoid follicles can be a benign finding.

    In light of these findings, her prognosis is fairly good. The median survival for treated follicular small cleaved lymphoma (follicular grade 1) can be up to 15 years. The initial treatment for advanced low-grade lymphoma is very controversial. It responds to both single- and multiple-agent chemotherapy as well as radiation therapy. However, regimens that include anti-CD20 antibody do seem to increase survival. Attempts to eradicate disease with high-dose chemotherapy (with or without bone marrow rescue) have not been shown to prolong overall survival. It would still not be unreasonable to wait until this patient becomes symptomatic before starting treatment.

  • What are the implications of her cytogenetic abnormalities?

    The t(14;18) abnormality is a common finding in the setting of follicular small cleaved lymphoma. In this abnormality, the bc12 oncogene on chromosome 18 has been translocated to the immunoglobulin heavy chain locus on chromosome 14. It is now widely accepted that this abnormality is found in virtually all patients with this histologic pattern. It is also found in approximately 30% of patients with diffuse lymphoma, and, in these cases, it probably represents a histologic transformation from follicular small cleaved lymphoma, and can therefore be considered a poor prognostic indicator. It is not a prognostic factor in the setting of follicular small cleaved lymphoma. The presence of CD20 surface antigen documents that this is a B-cell lymphoma and patients will probably respond to anti-CD20 antibody.

  • Is it necessary to further evaluate or treat her hypogammaglobulinemia?

    Hypogammaglobulinemia is occasionally found in association with low-grade lymphoma and more often chronic lymphocytic leukemia. It is not often a problem,

    P.321


    as it is in multiple myeloma, but this immunologic defect should be considered when infections occur in these patients or when neutropenia is precipitated by treatment or bone marrow involvement. Patients with life-threatening infections may benefit from gamma globulin therapy.

  • What is the significance of the monoclonal IgM spike in this patient?

    Although the size of the spike is not quantified, it is still important information because of the possibility of hyperviscosity associated with IgM, unlike IgG for which hyperviscosity is far less likely. Waldenstrom's macroglobulinemia is associated with lymphoma, and is usually seen in patients with diffuse small lymphocytic lymphoma. A monoclonal gammopathy can affect up to 15% of the patients with low-grade lymphoma and is most commonly seen when the cells have plasmacytoid features.

  • What form of therapy would you offer her when the painful and enlarging nodes are detected, and what outcome can she expect?

    The decision of when to treat low-grade non Hodgkin's lymphoma is, as already mentioned, a controversial issue. Most physicians recommend waiting until symptoms appear, consisting of rapidly enlarging nodes, B symptoms, cytopenias due to bone marrow involvement, or an increased adenopathy that threatens organ function.

    If the symptomatic disease is localized, radiation therapy that focuses on the site involved is a viable option. For the management of more generalized disease, a single alkylating agent with or without steroids can be very effective. More aggressive combination chemotherapy can also be considered, especially with anti-CD20 monoclonal antibody therapy.

    In determining the likely outcome of treatment in this patient, complete remissions can be achieved in the setting of low-grade lymphoma, mostly for localized (stage I and II) disease. Spontaneous remission can also occur (approximately 5% to 10%). The cure rate for advanced low-grade lymphoma is very low. Even with the institution of aggressive chemotherapy (see preceding text), less than 10% of affected patients remain disease free after 5 years.

  • How would you evaluate the anemia that develops with the alkylating agent therapy?

    During this patient's initial evaluation, she was noted to have a mild anemia with a slight increase in erythroid precursors. Her LDH level was elevated, but her liver enzyme values were normal. In this case, the raised LDH level could represent either a high turnover of tumor cells or the destruction of red blood cells (hemolysis), or both. The slight elevation in the number of red blood cells could also be due to peripheral destruction. This patient should undergo a complete assessment of her anemia, including evaluation for hemolysis; iron, cobalamin, and folate deficiency; and bone marrow involvement by lymphoma (an unlikely cause in this setting).

  • On the basis of the findings yielded by the investigations for her anemia, what are the treatment options at this point?

    This patient has hemolytic anemia. It is more commonly seen with chronic lymphocytic leukemia (the leukemia phase of follicular small cleaved lymphoma). Often patients have an underlying compensated hemolysis, as this patient did, with anemia, increased erythroids, and an increased LDH level. Beginning the treatment

    P.322


    with chemotherapy can unmask the hemolysis because the bone marrow response is retarded by marrow suppressive agents.

    The treatment for hemolytic anemia in this setting has two goals (a) to terminate the hemolytic process, and (b) to treat the underlying disease. Treatment with steroids may address both problems. If the hemolysis is not stopped with high-dose steroids, immunoglobulin therapy should be started. Once the hemolysis is controlled, a more aggressive chemotherapeutic regimen may be implemented.

  • With the appearance of increased splenomegaly, what is the differential diagnosis, and how should you confirm it?

    The increased splenomegaly that does not respond to the previously used chemotherapy may represent a transformation to a more aggressive histologic type of lymphoma. This occurs at a rate of approximately 5% per year in patients with follicular small cleaved lymphoma; usually, the transformation is to a diffuse large cell lymphoma. Histologic transformation represents a change in the natural history of the disease and signifies a much shortened survival.

    Histologic transformation should be documented by tissue examination. Again, a lymph node or mass is the best source of tissue for this purpose.

  • What are the treatment options in this patient whose disease is now in an advanced stage?

    When histologic transformation occurs, the prognosis is very poor and patients respond poorly to even the most aggressive chemotherapeutic regimens because of chemoresistant disease. In older patients who have concurrent disease, it may be reasonable to use only palliative measures (pain management) and perhaps administer local radiation therapy, if needed.

Suggested Readings

Bennett CL, Armitage JL, Armitage GO, et al. Costs of care and outcomes for high-dose therapy and autologous transplantation for lymphoid malignancies: results from the University of Nebraska 1987 through 1991. J Clin Oncol 1995;13:969.

Canellos G. Is there an effective salvage therapy for advanced Hodgkin's disease? Ann Oncol 1991;2:1.

DeVita VT Jr, Hubbard SM, Longo DL. Treatment of Hodgkin's disease. J Natl Cancer Inst 1990;10:19.

Hancock SL, Hoppe RT. Long-term complications of treatment and causes of mortality after Hodgkin's disease. Semin Radiat Oncol 1996;6:225.

Koh HK, Foss FM. Cutaneous T-cell lymphoma. Hematol Oncol Clin North Am 1995;9:943.

National Cancer Institute. Summary and description of a working formulation for clinical usage: the Non-Hodgkin's Lymphoma Pathologic Classification Project. Cancer 1982;49:2112.

Waldmann TA, Davis MM, Bongiovanni KF, et al. Rearrangements of genes for the antigen receptor on T cells as markers of lineage and clonality in human lymphoid neoplasms. N Engl J Med 1985;313:776.

Young RC, Longo DL, Glatstein E, et al. The treatment of indolent lymphomas: watchful waiting v aggressive combined modality treatment. Semin Hematol 1988;25:11.

P.323


Lung Cancer

  • What are the approximate incidence, death rate, and risk factors for lung cancer?

  • What are the two pathologic categories of lung cancer, and their histologic features?

  • What are some clinical features that may suggest a correlation with a certain pathologic subtype?

  • What is essentially the mainstay of curative therapy in non small cell lung cancer (NSCLC)?

  • How does the staging and treatment of small cell lung cancer (SCLC) differ from that of NSCLC?

  • What are the two major determinants of prognosis for both NSCLC and SCLC?

Discussion

  • What are the approximate incidence, death rate, and risk factors for lung cancer?

    Each year, lung cancer kills more men and women in the United States than any other cancer, and there are approximately 150,000 new cases of lung cancer diagnosed each year. At the time of diagnosis, only 35% of patients have local disease; therefore, the disease has spread to regional nodes or distant sites in 65%. However, even in patients with nonmetastatic (local) disease, complete cure is the exception; therefore, the yearly mortality rate approaches the annual incidence, and this was estimated to be 172,000 in 2005.

    Approximately 90% of all patients diagnosed with lung cancer have a history of smoking, and the causal relationship between tobacco use and lung cancer makes it a major public health problem and one of the most potentially preventable diseases. Other important risk factors account for less than 10% of cases of lung cancer diagnosed, and these include uranium and radon exposure and passive smoking (another reason for smoking cessation programs). The risk of acquiring lung cancer falls significantly in the first 5 years after the cessation of smoking, and, even after 20 years, the risk is higher than in people who have never smoked.

    To illustrate the seriousness of the public health problem, the lung cancer incidence between 1940 and 2000 has risen by 60% in women, from 7 to 45 per 100,000, and this is mainly due to the increased use of tobacco in the female population. In addition, a higher-than-expected incidence of lung cancer has been seen in women in the lesser pack-year categories, suggesting that women are acquiring lung cancer at a younger age and after smoking fewer years than men.

    Furthermore, although overall tobacco use is decreasing in the United States, smoking may be increasing among certain groups of minorities and adolescents, and recent tobacco company advertising campaigns have been

    P.324


    directed toward these groups. The years of potential lost life resulting from lung cancer mortality in these groups is probably twice that seen for the remaining population.

  • What are the two pathologic categories of lung cancer, and their histologic features?

    For both treatment and prognostic purposes, most lung cancers are divided into two clinically useful categories: SCLC, which accounts for 15% of all lung cancers, and NSCLC, which consists of squamous cell carcinomas (40% of the lung cancers), adenocarcinomas (40% of the lung cancers), and large cell carcinomas and others (5% of the lung cancers).

  • What are some clinical features that may suggest a correlation with a certain pathologic subtype?

    Patients may present with a variety of symptoms, including cough, hemoptysis, shortness of breath, chest pain, or unexplained weight loss (Table 7-12). Abnormalities revealed by the physical examination may suggest the diagnosis and include signs of lung consolidation resulting from an obstructed bronchus, supraclavicular adenopathy stemming from the local and regional spread of the cancer, or Horner syndrome, which is due to tumor impingement on the sympathetic nerve fibers that course near the apex of the lung.

    Laboratory examination may reveal the presence of hyponatremia due to the syndrome of inappropriate antidiuretic hormone secretion (SIADH); this is a paraneoplastic syndrome caused by inappropriate vasopressin secretion, and is seen most often in the setting of SCLC. The hyponatremia that presumably results from SIADH can be demonstrated in up to 60% of the patients with SCLC, by administering a water load. Cushing's syndrome may develop secondary to the excessive production of adrenocorticotropic hormone by the tumor, and, again, is most commonly seen in SCLC. Both the absolute and ionized serum calcium levels may be high, and this can be due to multiple reasons, including metastases to bone as well as the production of a parathyroid hormone like substance from the cancer. Hypercalcemia is most often seen

    P.325


    in patients with squamous (epidermoid) lung cancer, but may be associated with any histologic subtype. The location of the tumor by chest radiographic studies as well as specific laboratory findings can suggest certain histologic types. Squamous cell carcinomas and SCLCs tend to be found centrally on the chest radiography. Squamous cell lung cancer tends to cavitate, and this can be seen on chest radiographs. Adenocarcinoma tends to occur peripherally in the lung, and this is the most common lung cancer in those who have never smoked.

    Table 7-12 Signs and Symptoms of Lung Cancer

    Signs and Symptoms No. of Patients (%)
    Routine chest radiographic study (asymptomatic) 16
    Hemoptysis 30
    Cough 25
    Dyspnea 11
    Pneumonitis 8
    Pain 6
    Wheezing 2
    Dysphagia 1
    Hoarseness 0.5
    Systemic symptoms weight loss 0.5

    Although these clinically useful associations cannot establish a diagnosis, they can help the oncologist direct the nature of the evaluation, both before and after the histologic type is known.

  • What is essentially the mainstay of curative therapy in NSCLC?

    Significant cure rates for NSCLC are achieved only for complete surgical resection of nonmetastatic disease. The staging process defines the extent of disease spread, and the stage correlates with the resectability and hence with the cure rates and survival. No cures are seen using the current forms of chemotherapy, and cure is uncommon with radiation therapy alone (approximately 5% of stage III cases).

    In all cases in which resection is performed with curative intent, the decision to go ahead with resection is based on the expectation that no gross residual disease will remain at the conclusion of the procedure. This involves multiple preoperative decisions, including (a) verifying that the patient's lung function can tolerate obligate resection of some normal lung; (b) ensuring that there is no pulmonary or extrapulmonary disease that would preclude major surgery and general anesthesia; and (c) accurately staging the extent of disease to select only those patients with a reasonable chance of complete resection. Even with these considerations, however, overall only 20% of all patients with NSCLC survive 5 years, and less than half of patients who undergo successful resection remain free of disease at 5 years.

    At present, only stage I and II, and some stage III, patients are considered good candidates for resection with a reasonable chance of cure. This means the tumor cannot be associated with malignant effusion, cannot have spread to contralateral mediastinal lymph nodes, and cannot have metastasized to any distant site. The usual sites of distant metastases in patients with lung cancer include the adrenal glands, bone, brain, lung, lymph nodes, and pleura. Less common sites of metastases are the skin and bone marrow.

    The standard treatment for stage IIIB patients (i.e., those without distant metastases but with unresectable and locally advanced disease) is radiation therapy to a total dose of 55 to 60 Gy along with chemotherapy. Only 20% of these patients are alive at 5 years. Stage IV patients are those with distant metastases and, by and large, treatment in this setting is for palliation only and consists mostly of chemotherapy using platinum-based regimens.

  • How does the staging and treatment of SCLC differ from that of NSCLC?

    Small cell lung carcinoma differs markedly from the other pathologic types of lung cancer in terms of its natural history, cell biology, and response to therapy, and is distinct from NSCLC. SCLC has a rapid clinical course, and

    P.326


    the median survival in untreated patients is only 2 months for metastatic and 6 months for localized disease. On the basis of autopsy data, 80% of patients with SCLC have distant metastases at the time of diagnosis. This includes the 40% who at diagnosis have no evidence of distant metastases according to the results of the usual staging procedures.

    For practical reasons, SCLC is further defined as either limited or extensive. In limited disease, the tumor is confined to the hemithorax of origin and regional lymph nodes, and can be encompassed in a tolerable radiation therapy field. In extensive disease, tumor exists beyond these bounds, usually distant metastases. Surgery (other than for diagnostic biopsy) has no role in the management of SCLC because even most of the patients with limited disease have subclinical distant metastases; complete response rates to chemotherapy in SCLC are as high as 40% to 70%, with complete plus partial response rates as high as 80% to 85%, and survival in the setting of untreated disease is dismal. Instead, most patients should receive combination chemotherapy. With six or more courses of chemotherapy and radiation, approximately 20% of patients with limited disease can expect to be alive at 5 years, and these represent probable cures. The current standard of therapy includes irradiation of the involved areas as well as chemotherapy to decrease the chances of relapse, locally.

    In patients with extensive disease who undergo chemotherapy, the partial plus complete response rates can range from 50% to 85% and the median survival can range from 7 to 11 months; however, there are only anecdotal reports of cure at 5 years.

  • What are the two major determinants of prognosis for both NSCLC and SCLC?

    For both SCLC and NSCLC, the prognosis depends on (a) the tumor extent (graded as limited or extensive in SCLC and as stage I to IV in NSCLC) and (b) the performance status of the patient. Better responses to therapy and certainly more significant cure rates are seen in patients with lower-stage lung cancers. In addition, the performance status [usually measured by the Eastern Cooperative Oncology Group (ECOG) scale] is a major predictor of response to therapy and survival. Patients who are symptomatic or, more significantly, nonambulatory (as indicated by their performance status) are less likely to respond to therapy and have shorter survivals. The performance status scales most often used by clinicians are given in Table 7-13.

Case 1

A 56-year-old real estate broker with a 76 pack-year history of tobacco use (he has smoked two packs of cigarettes per day since 18 years of age) has been followed up regularly by his physician. He undergoes yearly chest radiographic studies, and the most recent radiographs obtained 8 months earlier were normal. He is seen by his physician because of 10 days of hemoptysis, consisting of blood-tinged sputum production, in the setting of a chronic cough. He denies weight loss, chest pain, and bone pain, and he experiences no increased dyspnea on exertion. On examination, his lungs are found to be clear; there is neither hepatosplenomegaly nor clubbing and the neurologic findings are grossly nonfocal. Laboratory studies show normal liver function, a calcium level of

P.327


11.1 mg/dL, and an albumin level of 3.9 g/dL. His complete blood count is normal. A chest radiographic study demonstrates a new, 2 3 cm, right hilar mass.

Table 7-13 Karnofsky and Zubrod Performance Status Scales

Definition Karnofsky Scale (%) (Old Complex Classification) Zubrod Scale (SWOG, ECOG are Variations)
Normal; no complaints; no evidence of disease 100 0
Able to carry on normal activity; minor signs or symptoms of disease 90 1
Normal activity with effort; some signs or symptoms of disease (no special care needed; fully ambulatory) 80 1
Cares for self but unable to carry on normal activity (unable to work; in bed <50%/d) 70 2
Requires occasional assistance but is able to care for most needs (in bed >50%/d but not bedridden) 60 2
Requires considerable assistance and frequent medical care 50 3
Disabled; requires special care and assistance 40 3
Bedridden 30 4
Very sick; hospitalization necessary 20 4
Moribund 10 4
Dead 0 5
SWOG, Southwest Oncology Group; ECOG, Eastern Cooperative Oncology Group.

  • What diagnostic tests should be performed in this patient?

  • Was a yearly chest radiographic study a reasonable screening procedure for this man who smokes, or should this have been done more frequently?

  • What clinical finding suggests (although does not establish) the histologic type of this patient's lung cancer, and what type is it?

  • What stage of lung cancer is this patient in, and what further studies should be performed?

  • If the patient has stage II NSCLC, what is the appropriate treatment?

  • What further measures can be taken to reduce the patient's risk of death from lung cancer?

Case Discussion

  • What diagnostic tests should be performed in this patient?

    Most lung cancers are diagnosed on the basis of the findings yielded by bronchoscopic biopsy of a lung mass. In some situations, however, there is an easier

    P.328


    source of tissue. For example, biopsy of an abnormal (palpable) supraclavicular lymph node, if present, will likely yield tissue adequate for histologic study as well as for staging purposes, and the information gained is important in planning treatment.

    The clinician must be wary of initial diagnoses of malignancy based on the findings revealed by the fine-needle aspiration (FNA) technique. This method obtains either only single cells or small clumps of cells for cytologic study within the aspirate, and it is almost always difficult or impossible to determine an exact pathologic diagnosis based on the information revealed. FNA biopsy should usually be reserved for confirmation of metastases or recurrences after an initial diagnosis has already been made. In most cases, bronchoscopic biopsy obtains tissue adequate for histologic examination.

    Sometimes, patients present with symptoms stemming from metastatic disease, and the source of a primary tumor is not as clear as it is in this patient. In such an event, correlation of the pathologic characteristics including special stains with the likely sources of the primary cancer can suggest the further radiologic and diagnostic tests to be performed to determine the origin of the tumor.

  • Was a yearly chest radiographic study a reasonable screening procedure for this man who smokes, or should this have been done more frequently?

    Because lung cancer (both NSCLC and SCLC) cure rates are significant only in the setting of earlier-stage disease (stage I and II NSCLC and limited-stage SCLC), it seems reasonable that earlier detection would increase the percentage of cures. In several randomized, controlled trials, mass screening using roentgenographic and sputum cytology has been done in 31,360 high-risk patients (men 45 years of age or older who smoked at least one pack of cigarettes per day). All these studies have shown that intensive screening can detect early lung cancer, but 45% to 60% of patients so identified actually have stage II or III disease, for whom the 5-year survival rate is 35%. Investigators at all three centers contend that the mortality rates for lung cancer do not differ significantly between the screening group and the control group. Therefore, at this time there is no justification for the large-scale application of these screening methods, even in high-risk populations.

    Yearly screening chest radiographic studies in the smoking population is not recommended and no study findings support more frequent screening chest roentgenography to decrease the mortality.

  • What clinical finding suggests (although does not establish) the histologic type of this patient's lung cancer, and what type is it?

    This patient's chest radiograph revealed the lung mass to be central (hilar) in location. The two common tumor types typically found in this location are squamous (epidermoid) cell carcinoma (a type of NSCLC) and SCLC, although biopsy findings are needed to establish the diagnosis. The high calcium value is most often seen with squamous cell carcinoma.

  • What stage of lung cancer is this patient in, and what further studies should be performed?

    According to the current American Joint Committee on Cancer (TNM) staging system, the stage of this patient's cancer is T2 NX MX (tumor 3 cm in greatest dimension, nodal status unknown, and unknown metastases).

    P.329


    In the setting of NSCLC, normal liver function test results (including aminotransferases, alkaline phosphatase, LDH, and -glutamyl transpeptidase) predict reasonably well that no liver metastases will be found on a radionuclide or CT scan. In this patient, a CT scan of the chest should be obtained to evaluate the hilar and mediastinal nodes, and the examination can easily be extended through the liver and adrenals. If the CT findings and the alkaline phosphatase level are normal, and because the patient has no bone pain, these would predict no bone metastases, making a bone scan unnecessary. If findings from a thorough neurologic examination performed by a consulting neurologist are normal, CT scanning of the head can be deferred. In this setting, the patient would clinically be in stage II, graded T2 N0 M0. A PET scan would better confirm this staging if surgery is considered (see preceding text).

  • If the patient has stage II NSCLC, what is the appropriate treatment?

    For stage II NSCLC, surgical excision (lobectomy or pneumonectomy) with intent of cure is the treatment of choice. If the arterial blood gas values, pulmonary function tests, electrocardiogram, and past medical history indicate that the patient has no excess risk for major surgery, a thoracic surgeon is then consulted. Because the patient may still have spread of cancer to the mediastinal or hilar nodes (and not seen on a CT or PET scan as being abnormal), which would make his cancer unresectable, mediastinoscopy might be performed especially if the patient is not the best surgical candidate.

    If these findings are negative, a right upper lobectomy through a lateral thoracotomy can be performed with subsequent pathologic study. If, for instance, a 2.8 3.2-cm tumor with two peribronchial nodes positive for squamous cell lung cancer is encountered at surgery, the patient is in pathologic stage II, designated T2 N1 M0, and his overall chance of 5-year survival ranges from 35% to 45% (with postoperative chemotherapy). If the patient is found to have a pathologic stage I tumor, the 5-year survival would approach 50% to 60%, mostly representing cures.

  • What further measures can be taken to reduce the patient's risk of death from lung cancer?

    Even patients surgically cured of their lung cancer have a significant chance of acquiring subsequent lung cancers, as well as other tumors. As with those in whom tumors have not yet developed, cessation of smoking can significantly reduce subsequent risk, and this patient should be strongly advised to do so.

Case 2

A 68-year-old woman presents to the emergency room because of a new-onset grand mal seizure. She is lethargic, but neurologic findings are otherwise normal. A head CT scan reveals a 2-cm right parietal and a 0.5-cm left occipital enhancing mass, and a chest radiographic study reveals a 4-cm left hilar mass with distal atelectasis. She has smoked one pack of cigarettes per day for 40 years, but quit 1 month ago. Anticonvulsants are administered, the patient is admitted to the hospital, and bronchoscopy is performed, which shows a mass in the right mainstem bronchus. Biopsy is done and pathologic examination reveals a small cell cancer.

A bone scan shows an abnormality in her left femur, and her alkaline phosphatase level is increased at 214 mU/mL. A CT scan of the abdomen is normal.

P.330


  • What stage is this patient's cancer?

  • How should this patient's cancer be treated?

  • If, 6 weeks after diagnosis, physical examination reveals only alopecia, and a chest radiograph and bone scan are normal, what is the likelihood of cure?

Case Discussion

  • What stage is this patient's cancer?

    The patient has extensive SCLC with metastases to the brain.

  • How should this patient's cancer be treated?

    Most chemotherapy traverses the blood brain barrier poorly. Therefore, the patient should be started on whole-brain radiation therapy. Because SCLC is a rapidly growing tumor, and radiation therapy lasts 4 to 6 weeks, combination chemotherapy should also be initiated.

  • If, 6 weeks after diagnosis, physical examination reveals only alopecia, and a chest radiograph and bone scan are normal, what is the likelihood of cure?

    Small cell lung cancer is responsive to both radiation therapy and chemotherapy. Complete responses are witnessed in 15% to 30% of patients with extensive disease. However, the 5-year survival is only 4% to 8% in this setting, and tumor usually recurs quickly.

Suggested Readings

Bunn PA, Lichter AS, Makuch RW, et al. Chemotherapy alone or chemotherapy with chest radiation therapy in limited stage small cell lung cancer: a prospective randomized trial. Ann Intern Med 1987;106:655.

Carney DN, de Leij L. Lung cancer biology. Semin Oncol 1988;15:199.

Elderly Lung Cancer Vinorelbine Italian Study Group. Effects of vinorelbine on quality of life and survival of elderly patients with advanced non-small-cell lung cancer. J Natl Cancer Inst 1999;91:66.

Farray D, Mirkovic N, Albain KS. Multimodality therapy for stage III non-small-cell lung cancer. J Clin Oncol 2005;23:3257.

Lababede O, Meziane MA, Rice TW. TNM staging of lung cancer: a quick reference chart. Chest 1999;115:233.

Mountain CF, Luckman JM, Hammer SP, et al. Lung cancer classification: the relationship of disease extent and cell type to survival in a clinical trials population. J Surg Oncol 1987;35:147.

Prostate Cancer

  • How common is prostate cancer?

  • How does prostate cancer arise and spread?

  • How is prostate cancer graded and staged, and why is this important?

  • P.331


  • What is the typical clinical presentation of prostate cancer?

  • After a physical examination, how is a suspicion of prostate cancer confirmed?

  • Once prostate cancer is diagnosed, how is a staging evaluation performed?

  • What treatment is recommended for stage Al prostate cancer?

  • Which category of patients is considered for a radical prostatectomy?

  • Is any alternative therapy available for patients with stage A2 or B prostate cancer?

  • What therapy may be administered to patients with stage C or D disease?

  • In terms of the recommended treatments, what survival can be expected in patients with stage A1, A2, B, or C prostate cancer?

  • What complications attend the use of radical prostatectomy and irradiation which patients should be aware of in advance?

  • What is the treatment for disseminated prostate cancer?

  • Is the prostate-specific antigen level useful in screening asymptomatic men for prostate cancer?

Discussion

  • How common is prostate cancer?

    Approximately 90,000 new cases of carcinoma of the prostate are identified every year. Of men in their eighties and older than 90 years, 50% and 90% of the prostates, respectively, are found to have carcinoma at autopsy. It is the second most common malignancy in American men and the most common cancerous cause of death in men older than 75 years. The clinical incidence is higher in blacks than in whites, and it is much lower for Japanese men living in Japan. Therefore, racial and environmental factors may be involved in its development.

  • How does prostate cancer arise and spread?

    Prostate adenocarcinoma arises from the epithelium of the peripheral acinar glands. It is slow growing, and half of all cases of early-stage (A) prostate cancer are found only at autopsy. Prostate cancer first extends locally through its capsule and then invades the lymphatics and blood vessels. Lymph node involvement occurs sequentially, with initial spread to the periprostatic and obturator nodes, and later the iliac and periaortic nodes. Distant hematogenous spread tends to occur late in the disease and frequently involves the axial and proximal skeleton, liver, and lungs. However, more than half of all symptomatic patients with prostate cancer present with metastases.

  • How is prostate cancer graded and staged, and why is this important?

    Prostate cancer is broadly staged into categories A, B, C, and D. Stage A is carcinoma detected by pathologic examination of glands removed because of clinically benign disease. Stage A1 represents well-differentiated tumor involving less than 5% of the surgical specimen, and stage A2 constitutes tumor either involving more than 5% of the surgical specimen or a poorly differentiated tumor. Stage B is clinically detectable disease that is intracapsular and involves one lobe (B1) or more than one lobe (B2). In stage A and B cancer, the prostate-specific antigen levels are elevated 15% and 25% of the instances,

    P.332


    respectively. Stage C is cancer that has invaded through the prostate capsule but is confined to the pelvis. Stage D, or metastatic disease, includes two categories: D1 (local pelvic nodal involvement) and D2 (distant, nodal, bone, or visceral disease), and is associated with elevated serum tumor markers.

  • What is the typical clinical presentation of prostate cancer?

    Classically, the disease is detected in elderly men with the occurrence of back pain, weight loss, anemia, urinary frequency, and nocturia, and, less often, dysuria, a slow urinary stream, urinary retention, and rarely hematuria.

    Prostate cancer can be found during a routine physical examination or screening, or when investigating for a pathologic fracture, bone pain, palpable lymphadenopathy, chronic renal insufficiency, anemia, cachexia, or a number of other seemingly unrelated signs and symptoms. Approximately 50% of the indurated prostatic lesions felt on rectal examination turn out to be adenocarcinoma on biopsy. The most effective screening test remains a careful rectal examination that should be performed starting from age 50.

  • After a physical examination, how is a suspicion of prostate cancer confirmed?

    Most often, the diagnosis of prostate cancer is confirmed by needle biopsy findings. The transrectal route has been made accurate and safe through the use of transrectal ultrasonography and the biopsy gun. Open biopsy and transurethral biopsy are less frequently used owing to the higher morbidity associated with their use.

  • Once prostate cancer is diagnosed, how is a staging evaluation performed?

    A thorough physical examination is performed to determine the size of the primary tumor and if any evidence of metastatic tumor is present. Serum prostate-specific antigen level and liver function are assessed. A chest radiographic study and radioisotope bone scan are routinely performed. Depending on the patient, pelvic or other CT scans, ultrasonography, bipedal or pelvic lymphadenectomy, and a skeletal radiographic survey may be required. Intravenous pyelography is performed to assess ureteral status.

    A complete blood count as well as BUN and serum creatinine measurements are routinely done to detect anemia or obstructive uropathy. The prostate-specific antigen level is sensitive (95%) and equally specific (>95%). A chest radiographic study may reveal metastases to the ribs, lungs, or hilar nodes. The bone scan is more sensitive and has largely supplanted the skeletal radiographic survey, but requires careful interpretation. The abdominal CT scan, together with positive FNA findings in the pelvic nodes, can often eliminate the need for a pelvic lymphadenectomy for staging purposes. The latter is not useful for stage A1 lesions, which almost never metastasize to pelvic nodes, or for poorly differentiated stage C prostate adenocarcinoma with elevated tumor markers, because more than 90% of these are metastatic. However, pelvic lymphadenectomy is frequently useful in staging intermediate disease to determine the prognosis and plan the treatment.

  • What treatment is recommended for stage A1 prostate cancer?

    Only observation is recommended by most urologists for patients with stage A1 prostate cancer because of the very slow and infrequent (<10%)

    P.333


    progression of the tumor at this stage. The data from follow-up studies have indicated that the survival for this group is similar to that of age-matched control subjects.

  • Which category of patients is considered for a radical prostatectomy?

    A radical prostatectomy is effective in patients with tumor confined to the prostate stages A2 and B carcinoma. The retropubic approach with a nerve-sparing technique to minimize the risk of impotence and/or incontinence is commonly used. This is usually preceded by bilateral pelvic node dissection and frozen-section examination to confirm noninvolvement by the tumor.

  • Is any alternative therapy available for patients with stage A2 or B prostate cancer?

    If a staging lymphadenectomy reveals no pelvic node metastases or if patients are poor operative risks, prostate cancer may be successfully treated with external-beam irradiation. In most centers, internal radiation combined with some external-beam radiation therapy is the treatment used. In general, the long-term control rate and survival for localized prostate cancer are similar regardless of whether surgery or irradiation is the therapy selected.

  • What therapy may be administered to patients with stage C or D disease?

    Patients with stage C or D disease often receive external-beam irradiation delivered to the pelvic and periaortic nodes, but this has not been observed consistently and clearly to enhance survival. In some symptomatic patients who have pelvic pain, lower extremity edema, hematuria, or urethral obstruction, palliative external-beam radiation therapy may be used. An alternative is to proceed directly to hormonal ablation therapy.

  • In terms of the recommended treatments, what survival can be expected in patients with stage A1, A2, B, or C prostate cancer?

    Only 1% of patients with stage A1 disease die of prostate cancer, and metastases develop in only 5% within 5 to 10 years without treatment. The natural history of this group is determined in the elderly by coexistent disease.

    Without treatment, 35% of patients in stage A2 have metastases and 20% die within 5 to 10 years of diagnosis. The results of radical prostatectomy closely approximate the clinical picture seen in untreated patients with stage A1 disease. The same applies to stage B1 and B2 disease, and hence it may be concluded that, with appropriate treatment, survival in patients with early-stage prostate cancer is similar to that of age-matched healthy control subjects. Therefore, most patients with stage A or B disease, particularly those older than 65 years, die of other causes.

  • What complications attend the use of radical prostatectomy and irradiation, which patients should be aware of in advance?

    Radical prostatectomy, even using modern nerve-sparing techniques, can cause impotence in 20% to 50% of patients and urinary incontinence in 5%. Radiation therapy is associated with symptoms of radiation proctitis and cystourethritis. None of the immediate postoperative complications occurs, but radiation therapy has a similar incidence of the other side effects, although their onset may take longer.

  • P.334


  • What is the treatment for disseminated prostate cancer?

    Hormonal manipulation is the standard approach for the treatment of disseminated prostate cancer. Chemotherapy is minimally effective and is considered only if hormonal treatment fails (hormone refractory cancer).

    The response to treatment in the setting of prostate cancer is typically difficult to assess. It includes physical findings, performance status, bone pain, weight change, and the hemoglobin level. Bone scans and other imaging techniques and, most important, prostate-specific antigen levels also predict response.

    Orchiectomy leads directly to low testosterone levels. Luteinizing hormone-releasing hormone agonists (leuprolide and goserelin) bring about suppressed testosterone levels in 2 to 3 weeks and can be administered conveniently at monthly intervals or longer as a subcutaneous depot preparation. Testosterone receptor level antagonism can be achieved by the more recently introduced nonsteroidal agent, flutamide.

    Choosing among the commonly used initial hormonal approaches (i.e., leuprolide or flutamide) often depends on individual patient preference with respect to toxicities and cost.

    The combination of a luteinizing hormone-releasing hormone agonist and an antiandrogen may bring about an increased response rate in patients with low-volume stage D1 prostate cancer. However, in general, no one hormonal treatment has been conclusively proved to be superior in terms of response. Approximately 10% of patients experience complete or partial responses and, in another 20%, the disease remains stable for at least 3 months. The treatment of metastatic prostate cancer doses prolong survival but there are no definitive cures. If initial hormonal manipulation fails, a second hormonal treatment achieves a response in only 10% to 20% of patients.

    Chemotherapy achieves responses (usually partial) in 40% of patients; in addition, it can often produce toxicity but does prolong survival in terms of months. Patients who fail initial therapy are usually offered palliative or experimental chemotherapy approaches. Some evidence now exists for the efficacy of suramin, which blocks growth factor effects in prostate cancer.

  • Is the prostate-specific antigen level useful in screening asymptomatic men for prostate cancer?

    An elevated prostate-specific antigen value is useful in the detection of early prostate cancer in asymptomatic men. However, because early prostate cancer is often not associated with clinical disease, studies examining the utility of mass prostate-specific antigen screening are still under way to determine if this screening ultimately improves survival. Most clinicians agree that a proper prostate examination on rectal evaluation for men older than 50 years is a cost-effective and potentially helpful screening method.

Case

A 65-year-old man presents to the emergency room because of a backache that has lasted for several days, which became severe after a fall. Six years before, the patient

P.335


began to have urinary frequency and dribbling on micturition, and was noted to have a hard, nontender prostate nodule with obliteration of the lateral sulcus and a palpable seminal vesicle on one side, for which he received external-beam irradiation. He has been on leuprolide injections since that time. His prostate-specific antigen level was slightly elevated before therapy but returned to normal thereafter, and was normal 4 months ago.

Physical examination reveals a diffuse tenderness over the lower thoracic and lumbar spines. Rectal examination reveals a hard, irregular prostate. The remainder of the physical examination findings are unremarkable. Laboratory workup shows the following: hemoglobin, 11.5 g/dL; hematocrit, 32%; white blood cell count, 9.8 109/L; platelets, 162 109/L; white blood cell differential neutrophils, 68%; lymphocytes, 26%; monocytes, 4%; band forms, 2%; erythrocyte sedimentation rate, 87 mm in the first hour; and alkaline phosphatase, 320 U (normal, up to 150 U). Two nucleated red blood cells are seen per 100 white blood cells.

A chest radiographic study shows increased bone densities in several ribs, and a radiograph of the lumbar spine shows multiple areas of bone sclerosis but no fractures. A bone scan shows multiple areas of increased activity scattered over the axial skeleton.

  • What was the stage of this patient's prostate cancer 3 years ago?

  • Was the treatment given at that time appropriate?

  • How would you go about proving a diagnosis of prostate cancer now?

  • What are the treatment options and prognosis in this patient now?

  • What are the complications of external-beam irradiation for the treatment of stage C prostate cancer?

Case Discussion

  • What was the stage of this patient's prostate cancer 3 years ago?

    The patient had stage C prostate cancer 6 years ago based on palpable extension of the tumor across the prostatic capsule and a negative bone scan. A palpable seminal vesicle is abnormal.

  • Was the treatment given at that time appropriate?

    The treatment for stage C prostate cancer is external-beam irradiation, usually with internal radiation or, in selected instances, radical prostatectomy. Adjuvant hormonal treatment does prolong time to recurrence and probably improves survival.

  • How would you go about proving a diagnosis of prostate cancer now?

    The presence of immature white and red blood cells in the circulation together with the anemia (a leukoerythroblastic picture) suggests the existence of bone marrow infiltration. A bone marrow biopsy and staining for acid phosphatase and prostate-specific antigen might clinch the diagnosis. An elevated serum prostate-specific antigen level in the presence of sclerotic bone lesions establishes the diagnosis of metastatic prostate cancer.

  • What are the treatment options and prognosis in this patient now?

    The treatment option for metastatic prostate cancer with predominant bone lesions is to change to hormonal therapy. Radiation therapy can be used for the management of painful bone lesions not amenable to androgen deprivation. A

    P.336


    change in androgen deprivation therapy can be effected by blocking androgen receptor function. This change gives a median time to further progression of disease of approximately 15 months and a median survival of 30 months. Bilateral orchiectomy is an option but may not be acceptable to some patients for psychological reasons.

  • What are the complications of external-beam irradiation for the treatment of stage C prostate cancer?

    The common complications of external-beam irradiation are impotence, suprapubic or perineal edema, proctitis, persistent tumor, and fibrosis of the prostate, making it hard and its substance clinically indistinguishable from that of malignancy.

Suggested Readings

Albertsen PC, Fryback DG, Storer BE, et al. Long-term survival among men with conservatively treated localized prostate cancer. JAMA 1995;274:626.

Carvalhal GF, Smith DS, Mager DE, et al. Digital rectal examination for detecting prostate cancer at prostate specific antigen levels of 4 ng/mL or less. J Urol 1999;161:835.

Crawford ED, Eisenberger MA, McLeod DG, et al. A controlled trial of leuprolide with and without flutamide in prostatic carcinoma. N Engl J Med 1989;321:419.

George NJR. Natural history of localized prostatic cancer managed by conservative therapy alone. Lancet 1988;1:494.

Gerber GS, Chodak GW. Routine screening for cancer of the prostate. J Natl Cancer Inst 1991;83:329.

Gleason DF. Histologic grade, clinical stage, and patient age in prostate cancer. NCI Monogr 1988;7:15.

Gwede CK, Pow-Sang J, Seigne J, et al. Treatment decision-making strategies and influences in patients with localized prostate carcinoma. Cancer 2005;104:1381.



Internal Medicine Casebook. Real Patients, Real Answers
The Internal Medicine Casebook: Real Patients, Real Answers
ISBN: 0781765293
EAN: 2147483647
Year: 2007
Pages: 14

flylib.com © 2008-2017.
If you may any questions please contact us: flylib@qtcs.net